You are on page 1of 49

Study Notes – Family Medicine & Physical ExamJames Lamberg 28Jul2010

Textbooks, Family Medicine: Essential Family Medicine, First Aid for Family Medicine Boards
Textbooks, Clinical Skills: Swartz Physical Diagnosis, Physical Diagnosis Secrets, OMT Review
--------------------------------------------------------------------------------------------------------------------------------------------
Common Problems in Family Medicine
Abdominal: Peptic Ulcer Disease, Cholecystitis, Constipation/Diarrhea, Irritable Bowel Syndrome, Gastritis
Aches/Pains: Back Pain, Neck Pain, Shoulder Pain, Sprains/Strains, Bursitis, Osteoarthritis, Gout
CAD: Angina, Congestive Heart Failure
Chest Pain: Costochondritis (Tietze Syndrome), Gastroesophageal Reflux/Hiatal Hernia
Dysuria: Urethritis, Cystitis, Pyelonephritis, STD, Prostatitis
General: Depression/Anxiety, Diabetes Type II (NIDDM), Hypertension, Metabolic Syndrome
Gynecologic: Incontinence, Menopausal Syndromes, Menstrual Disorders, PID, Vaginitis, Cervicitis, Family
Planning, Contraception
Headache: Tension, Migraine
Preventative: Immunizations, Weight Management/Exercise, Smoking Cessation, School/Sports/Occupational
Physical, Well Child Assessment, Well Adult Assessment, Women’s Health Assessment
Lower RTI: Pneumonia, Bronchitis
Skin Problems: Dermatitis, Eczema, Common Rashes/Viral Exanthems, Common Benign Skin Lesions, Common
Dermatological Malignancies
SOB: Emphysema, COPD, Asthma
Substances: Tobacco, Alcohol, Prescription Drugs, Illicit Drugs
Upper RTI: Allergic Rhinitis, Otitis Media, Pharyngitis/Sore Throat, Sinusitis, Viral URI
Vomiting: Gastroenteritis
Violence: Family Violence, Sexual Violence, Child Abuse, Senior Abuse
Miscellaneous: Medication Compliance, Anticipatory Guidance/Family Life Cycle Issues, Referral and Consultation
--------------------------------------------------------------------------------------------------------------------------------------------
Emergency Medicine Rapid Review – First Aid For USMLE Step 2 (3rd, Le)
Class of drugs that may cause syndrome of muscle Antipsychotics (neuroleptic malignant syndrome).
rigidity, hyperthermia, autonomic instability, and
extrapyramidal symptoms.
Side effects of corticosteroids. Acute mania, immunosuppression, thin skin,
osteoporosis, easy bruising, myopathies.
Treatment for DTs. Benzodiazepines.
Treatment for acetaminophen overdose. N-acetylcysteine.
Treatment for opioid overdose. Naloxone.
Treatment for benzodiazepine overdose. Flumazenil.
Treatment for neuroleptic malignant syndrome and Dantrolene.
malignant hyperthermia.
Treatment for malignant hypertension. Nitroprusside.
Treatment of atrial fibrillation. Rate control, rhythm conversion, and anticoagulation.
Treatment of supraventricular tachycardia. If stable, rate control with carotid massage or other
vagal stimulation; if unsuccessful, consider adenosine.
Causes of drug-induced SLE. INH, penicillamine, hydralazine, procainamide,
chlorpromazine, methyldopa, quinidine.
Macrocytic, megaloblastic anemia with neurologic B12 deficiency.
symptoms.
Macrocytic, megaloblastic anemia without neurologic Folate deficiency.
symptoms.
A burn patient presents with cherry-red flushed skin Treat CO poisoning with 100% O2 or with hyperbaric
and coma. SaO2 is normal, but carboxyhemoglobin is O2 if poisoning is severe or the patient is pregnant.
elevated. Treatment?
Blood in the urethral meatus or high-riding prostate. Bladder rupture or urethral injury.
Test to rule out urethral injury. Retrograde cystourethrogram.
The most common organism in burn-related infections. Pseudomonas.
Method of calculating fluid repletion in burn patients. Parkland formula.

DO NOT DISTRIBUTE -1-


Study Notes – Family Medicine & Physical ExamJames Lamberg 28Jul2010

Radiographic indications for surgery in patients with Free air under the diaphragm, extravasation of contrast,
acute abdomen. severe bowel distention, space-occupying lesion (CT),
mesenteric occlusion (angiography).
Acceptable urine output in a trauma patient. 50 cc/hr.
Acceptable urine output in a stable patient. 30 cc/hr.
Cannon “a” waves. Third-degree heart block.
Signs of neurogenic shock. Hypotension and bradycardia.
Signs of ↑ ICP (Cushing’s triad). Hypertension, bradycardia, and abnormal respirations.
↓ CO, ↓ pulmonary capillary wedge pressure (PCWP), Hypovolemic shock.
↑ peripheral vascular resistance (PVR).
↓ CO, ↑ PCWP, ↑ PVR. Cardiogenic (or obstructive) shock.
↑ CO, ↓ PCWP, ↓ PVR. Septic or anaphylactic shock.
Treatment of septic shock. Fluids and antibiotics.
Treatment of cardiogenic shock. Identify cause; pressors (e.g., dopamine).
Treatment of hypovolemic shock. Identify cause; fluid and blood repletion.
Treatment of anaphylactic shock. Diphenhydramine or epinephrine 1:1000.
Supportive treatment for ARDS. Continuous positive airway pressure.
Signs of air embolism. A patient with chest trauma who was previously stable
suddenly dies.
Trauma series. AP chest, AP/lateral C-spine, AP pelvis.
--------------------------------------------------------------------------------------------------------------------------------------------
Cardiovascular Rapid Review – First Aid For USMLE Step 2 (3rd, Le)
Classic ECG finding in atrial flutter. “Sawtooth” P waves.
Definition of unstable angina. Angina is new, is worsening, or occurs at rest.
Antihypertensive for a diabetic patient with proteinuria. ACEI.
Beck’s triad for cardiac tamponade. Hypotension, distant heart sounds, and JVD.
Drugs that slow AV node transmission. β-blockers, digoxin, calcium channel blockers.
Hypercholesterolemia treatment that leads to flushing Niacin.
and pruritus.
Murmur: hypertrophic obstructive cardiomyopathy Systolic ejection murmur heard along the lateral sternal
(HOCM). border that ↑ with Valsalva maneuver and standing.
Murmur: aortic insufficiency. Diastolic, decrescendo, high-pitched, blowing murmur
that is best heard sitting up; ↑ with ↓ preload (handgrip
maneuver).
Murmur: aortic stenosis. Systolic crescendo/decrescendo murmur that radiates to
the neck; ↑ with ↑ preload (Valsalva maneuver).
Murmur: mitral regurgitation. Holosystolic murmur radiating axillae or carotids.
Murmur: mitral stenosis. Diastolic, mid- to late, low-pitched murmur.
Treatment for atrial fibrillation and atrial flutter. If unstable, cardiovert. If stable or chronic, rate control
with calcium channel blockers or β-blockers.
Treatment for ventricular fibrillation. Immediate cardioversion.
Autoimmune problem occurring 2-4 weeks post-MI. Dressler’s syndrome: fever, pericarditis, ↑ ESR.
IV drug use with JVD and holosystolic murmur at the Treat existing heart failure and replace the tricuspid
left sternal border. Treatment? valve.
Diagnostic test for hypertrophic cardiomyopathy. Echocardiogram (showing thickened left ventricular
wall and outflow obstruction).
A fall in systolic BP of > 10 mmHg with inspiration. Pulsus paradoxus (seen in cardiac tamponade).
Classic ECG findings in pericarditis. Low-voltage, diffuse ST-segment elevation.
Definition of hypertension. BP > 140/90 on three separate occasions 2 weeks apart.

DO NOT DISTRIBUTE -2-


Study Notes – Family Medicine & Physical ExamJames Lamberg 28Jul2010

Eight surgically correctable causes of hypertension. Renal artery stenosis, coarctation of the aorta,
pheochromocytoma, Conn’s syndrome, Cushing’s
syndrome, unilateral renal parenchymal disease,
hyperthyroidism, hyperparathyroidism.
Evaluation of a pulsatile abdominal mass and bruit. Abdominal ultrasound and CT.
Indications for surgical repair of abdominal aortic > 5.5 cm, rapidly enlarging, symptomatic, or ruptured.
aneurysm.
Treatment for acute coronary syndrome. Morphine, O2, sublingual nitroglycerin, ASA, IV β-
blockers, heparin.
What is metabolic syndrome? Abdominal obesity, high triglycerides, low HDL,
hypertension, insulin resistance, prothrombotic or
proinflammatory states.
Appropriate diagnostic test? A 50-year-old man with Exercise stress treadmill with ECG.
angina can exercise to 85% of maximum predicted
heart rate.
Appropriate diagnostic test? A 65-year-old woman Pharmacologic stress test (e.g., dobutamine echo).
with left bundle branch block and severe osteoarthritis
has unstable angina.
Target LDL in a patient with diabetes. < 70.
Signs of active ischemia during stress testing. Angina, ST-segment changes on ECG, or ↓ BP.
ECG findings suggesting MI. ST-segment elevation (depression means ischemia),
flattened T waves, and Q waves.
Coronary territories in MI. Anterior wall (LAD/diagonal), inferior (PDA),
posterior (left circumflex/oblique, RCA/marginal),
septum (LAD/diagonal).
A young patient has angina at rest with ST-segment Prinzmetal’s angina.
elevation. Cardiac enzymes are normal.
Common symptoms associated with silent Mls. CHF, shock, and altered mental status.
The diagnostic test for pulmonary embolism. V/Q scan.
An agent that reverses the effects of heparin. Protamine.
The coagulation parameter affected by warfarin. PT.
A young patient with a family history of sudden death Hypertrophic cardiomyopathy.
collapses and dies while exercising.
Endocarditis prophylaxis regimens. Oral surgery—amoxicillin; GI or GU procedures—
ampicillin and gentamicin before and amoxicillin after.
The 6 P’s of ischemia due to peripheral vascular Pain, pallor, pulselessness, paralysis, paresthesia,
disease. poikilothermia.
Virchow’s triad. Stasis, hypercoagulability, endothelial damage.
The most common cause of HTN in young women. OCPs.
The most common cause of HTN in young men. Excessive EtOH.
--------------------------------------------------------------------------------------------------------------------------------------------
Dermatology Rapid Review – First Aid For USMLE Step 2 (3rd, Le)
“Stuck-on” appearance. Seborrheic keratosis.
Red plaques with silvery-white scales, sharp margins. Psoriasis.
The most common type of skin cancer; the lesion is a Basal cell carcinoma.
pearly-colored papule with a translucent surface and
telangiectasias.
Honey-crusted lesions. Impetigo.
A febrile patient with a history of diabetes presents Cellulitis.
with a red, swollen, painful lower extremity.
Positive Nikolsky’s sign. Pemphigus vulgaris.
Negative Nikolsky's sign. Bullous pemphigoid.

DO NOT DISTRIBUTE -3-


Study Notes – Family Medicine & Physical ExamJames Lamberg 28Jul2010

A 55-year-old obese patient presents with dirty, velvety Acanthosis nigricans. Check fasting blood glucose to
patches on the back of the neck. rule out diabetes.
Dermatomal distribution. Varicella zoster.
Flat-topped papules. Lichen planus.
Iris-like target lesions. Erythema multiforme.
A lesion characteristically occurring in a linear pattern Contact dermatitis.
in areas where skin contacts clothing or jewelry.
Presents with a herald patch, Christmas-tree pattern. Pityriasis rosea.
A 16-year-old presents with an annular patch of Alopecia areata (an autoimmune process).
alopecia with broken-off, stubby hairs.
Pinkish, scaling, flat lesions on the chest and back; Pityriasis versicolor.
KOH prep has a “spaghetti-and-meatballs” appearance.
Four characteristics of a nevus suggestive of Asymmetry, border irregularity, color variation, and
melanoma. large diameter.
A premalignant lesion from sun exposure that can lead Actinic keratosis.
to squamous cell carcinoma.
“Dewdrops on a rose petal.” Lesions of 1° varicella.
“Cradle cap.” Seborrheic dermatitis. Treat with antifungals.
Associated with Propionibacterium acnes and changes Acne vulgaris.
in androgen levels.
A painful, recurrent vesicular eruption of Herpes simplex.
mucocutaneous surfaces.
Inflammation and epithelial thinning of the anogenital Lichen sclerosus.
area, predominantly in postmenopausal women.
Exophytic nodules on the skin with varying degrees of Squamous cell carcinoma.
scaling or ulceration; the second most common type of
skin cancer.
--------------------------------------------------------------------------------------------------------------------------------------------
Endocrinology Rapid Review – First Aid For USMLE Step 2 (3rd, Le)
The most common cause of hypothyroidism. Hashimoto’s thyroiditis.
Lab findings in Hashimoto’s thyroiditis. High TSH, low T4, antimicrosomal antibodies.
Exophthalmos, pretibial myxedema, and ↓ TSH. Graves’ disease.
The most common cause of Cushing’s syndrome. Iatrogenic corticosteroid administration. The second
most common cause is Cushing’s disease.
A patient presents with signs of hypocalcemia, high Hypoparathyroidism.
phosphorus, and low PTH.
“Stones, bones, groans, psychiatric overtones.” Signs and symptoms of hypercalcemia.
A patient complains of headache, weakness, and 1° hyperaldosteronism (due to Conn’s syndrome or
polyuria; exam reveals hypertension and tetany. Labs bilateral adrenal hyperplasia).
reveal hypernatremia, hypokalemia, and metabolic
alkalosis.
A patient presents with tachycardia, wild swings in BP, Pheochromocytoma.
headache, diaphoresis, altered mental status, and a
sense of panic.
Should α- or β-antagonists be used first in treating α-antagonists (phentolamine and phenoxybenzamine).
pheochromocytoma?
A patient with a history of lithium use presents with Nephrogenic diabetes insipidus (DI).
copious amounts of dilute urine.
Treatment of central DI. Administration of DDAVP ↓ serum osmolality and free
water restriction.
A postoperative patient with significant pain presents SIADH due to stress.
with hyponatremia and normal volume status.

DO NOT DISTRIBUTE -4-


Study Notes – Family Medicine & Physical ExamJames Lamberg 28Jul2010

An antidiabetic agent associated with lactic acidosis. Metformin.


A patient presents with weakness, nausea, vomiting, 1° adrenal insufficiency (Addison’s disease). Treat
weight loss, and new skin pigmentation. Labs show with replacement glucocorticoids, mineralocorticoids,
hyponatremia and hyperkalemia. Treatment? and IV fluids.
Goal HbA1c for a patient with DM. < 7.0.
Treatment of DKA. Fluids, insulin, and aggressive replacement of
electrolytes (e.g., K+).
Why are β-blockers contraindicated in diabetics? They can mask symptoms of hypoglycemia.
--------------------------------------------------------------------------------------------------------------------------------------------
Epidemiology Rapid Review – First Aid For USMLE Step 2 (3rd, Le)
Bias introduced into a study when a clinician is aware Observational bias.
of the patient’s treatment type.
Bias introduced when screening detects a disease Lead-time bias.
earlier and thus lengthens the time from diagnosis to
death.
If you want to know if geographical location affects Confounding variable.
infant mortality rate but most variation in infant
mortality is predicted by socioeconomic status, then
socioeconomic status is a _____.
The number of true positives divided by the number of Sensitivity.
patients with the disease is _____.
Sensitive tests have few false negatives and are used to Out.
rule _____ a disease.
PPD reactivity is used as a screening test because most Highly sensitive for TB.
people with TB (except those who are anergic) will
have a +PPD. Highly sensitive or specific?
Chronic diseases such as SLE—higher prevalence or Higher prevalence.
incidence?
Epidemics such as influenza—higher prevalence or Higher incidence.
incidence?
Cross-sectional survey—incidence or prevalence? Prevalence.
Cohort study—incidence or prevalence? Incidence and prevalence.
Case-control study—incidence or prevalence? Neither.
Describe a test that consistently gives identical results, High reliability, low validity.
but the results are wrong.
Difference between a cohort and a case-control study. Cohort studies can be used to calculate relative risk
(RR), incidence, and/or odds ratio (OR). Case-control
studies can be used to calculate an OR.
Attributable risk? The incidence rate (IR) of a disease in exposed – the IR
of a disease in unexposed.
Relative risk? The IR of a disease in a population exposed to a
particular factor ÷ the IR of those not exposed.
Odds ratio? The likelihood of a disease among individuals exposed
to a risk factor compared to those who have not been
exposed.
Number needed to treat? 1 ÷ (rate in untreated group – rate in treated group).
In which patients do you initiate colorectal cancer Patients with IBD; those with familial adenomatous
screening early? polyposis (FAP)/hereditary nonpolyposis colorectal
cancer (HNPCC); and those who have first-degree
relatives with adenomatous polyps (< 60 years of age)
or colorectal cancer.
The most common cancer in men and the most Prostate cancer is the most common cancer in men, but
common cause of death from cancer in men. lung cancer causes more deaths.

DO NOT DISTRIBUTE -5-


Study Notes – Family Medicine & Physical ExamJames Lamberg 28Jul2010

The percentage of cases within one SD of the mean? 68%, 95.4%, 99.7%.
Two SDs? Three SDs?
Birth rate? Number of live births per 1000 population in one year.
Fertility rate? Number of live births per 1000 females (15–44 years of
age) in one year.
Mortality rate? Number of deaths per 1000 population in one year.
Neonatal mortality rate? Number of deaths from birth to 28 days per 1000 live
births in one year.
Postnatal mortality rate? Number of deaths from 28 days to one year per 1000
live births in one year.
Infant mortality rate? Number of deaths from birth to one year of age per
1000 live births (neonatal + postnatal mortality) in one
year.
Fetal mortality rate? Number of deaths from 20 weeks’ gestation to birth per
1000 total births in one year.
Perinatal mortality rate? Number of deaths from 20 weeks’ gestation to one
month of life per 1000 total births in one year.
Maternal mortality rate? Number of deaths during pregnancy to 90 days
postpartum per 100,000 live births in one year.
--------------------------------------------------------------------------------------------------------------------------------------------
Ethics Rapid Review – First Aid For USMLE Step 2 (3rd, Le)
True or false: Once patients sign a statement giving False. Patients may change their minds at any time.
consent, they must continue treatment. Exceptions to the requirement of informed consent
include emergency situations and patients without
decision-making capacity.
A 15-year-old pregnant girl requires hospitalization for No. Parental consent is not necessary for the medical
preeclampsia. Is parental consent required? treatment of pregnant minors.
Doctor refers patient for an MRI at a facility she owns. Conflict of interest.
Involuntary psychiatric hospitalization can be The patient is a danger to self, a danger to others, or
undertaken for which three reasons? gravely disabled (unable to provide for basic needs).
True or false: Withdrawing a nonbeneficial treatment is True.
ethically similar to withholding a nonindicated one.
When can a physician refuse to continue treating a When there is no rationale for treatment, maximal
patient on the grounds of futility? intervention is failing, a given intervention has already
failed, and treatment will not achieve the goals of care.
An 8yo child is in a serious accident. She requires Treat immediately. Consent is implied in emergency
emergent transfusion, but her parents are not present. situations.
A son asks that his mother not be told about her A physician can withhold information from the patient
recently discovered cancer. only in the rare case of therapeutic privilege or if the
patient requests not to be told. A patient’s family
cannot require the physician to withhold information
from the patient.
Conditions in which confidentiality must be Real threat of harm to third parties; suicidal intentions;
overridden. certain contagious diseases; elder and child abuse.
Involuntary commitment or isolation for medical When treatment noncompliance represents a serious
treatment may be undertaken for what reason? danger to public health (e.g., active TB).
A 10-year-old child presents in status epilepticus, but Treat because the disease represents an immediate
her parents refuse treatment on religious grounds. threat to the child’s life. Then seek a court order.
--------------------------------------------------------------------------------------------------------------------------------------------
Gastrointestinal Rapid Review – First Aid For USMLE Step 2 (3rd, Le)
The most likely cause of acute lower GI bleed in Diverticulosis.
patients > 40 years of age.

DO NOT DISTRIBUTE -6-


Study Notes – Family Medicine & Physical ExamJames Lamberg 28Jul2010

A patient presents with sudden onset of severe, diffuse Emergent laparotomy to repair perforated viscus.
abdominal pain. Exam reveals peritoneal signs, and
AXR reveals free air under the diaphragm.
Management?
Diagnostic modality used when ultrasound is equivocal HIDA scan.
for cholecystitis.
Risk factors for cholelithiasis. Fat, female, fertile, forty, flatulent.
Inspiratory arrest during palpation of the RUQ. Murphy’s sign, seen in acute cholecystitis.
The most common cause of SBO in patients with no Hernia.
history of abdominal surgery.
The most common cause of SBO in patients with a Adhesions.
history of abdominal surgery.
Diarrhea: Most common organism Campylobacter
Diarrhea: Recent antibiotic use Clostridium difficile
Diarrhea: Camping Giardia
Diarrhea: Traveler's ETEC
Diarrhea: Church picnics/mayonnaise S. aureus
Diarrhea: Uncooked hamburgers E. coli O157:H7
Diarrhea: Fried rice Bacillus cereus
Diarrhea: Poultry/eggs Salmonella
Diarrhea: Raw seafood Vibrio, HAV
Diarrhea: AIDS Isospora, Cryptosporidium, Mycobacterium avium
complex (MAC)
Diarrhea: Pseudoappendicitis Yersinia
A 25-year-old Jewish man presents with pain and Crohn’s disease.
watery diarrhea after meals. Exam shows fistulas
between the bowel and skin and nodular lesions on his
tibias.
Inflammatory disease of the colon with ↑ risk of colon Ulcerative colitis (greater risk than Crohn’s).
cancer.
Extraintestinal manifestations of IBD. Uveitis, ankylosing spondylitis, pyoderma
gangrenosum, erythema nodosum, 1° sclerosing
cholangitis.
Medical treatment for IBD. 5-ASA agents and steroids during acute exacerbations.
Difference between Mallory-Weiss and Boerhaave Mallory-Weiss—superficial tear in the esophageal
tears. mucosa; Boerhaave—full-thickness esophageal
rupture.
Charcot’s triad. RUQ pain, jaundice, and fever/chills in the setting of
ascending cholangitis.
Reynolds’ pentad. Charcot’s triad plus shock and mental status changes,
with suppurative ascending cholangitis.
Medical treatment for hepatic encephalopathy. ↓ protein intake, lactulose, rifaximin.
First step in the management of a patient with an acute Establish the ABCs.
GI bleed.
A four-year-old child presents with oliguria, petechiae, Hemolytic-uremic syndrome (HUS) due to E. coli
and jaundice following an illness with bloody diarrhea. O157: H7.
Most likely diagnosis and cause?
Post-HBV exposure treatment. HBV immunoglobulin.
Classic causes of drug-induced hepatitis. TB medications (INH, rifampin, pyrazinamide),
acetaminophen, and tetracycline.
Hernia with highest risk of incarceration—indirect, Femoral hernia.
direct, or femoral?

DO NOT DISTRIBUTE -7-


Study Notes – Family Medicine & Physical ExamJames Lamberg 28Jul2010

A 40-year-old obese woman with elevated alkaline Biliary tract obstruction.


phosphatase, elevated bilirubin, pruritus, dark urine,
and clay-colored stools.
A 50-year-old man with a history of alcohol abuse Confirm the diagnosis of acute pancreatitis with
presents with boring epigastric pain that radiates to the elevated amylase and lipase. Make the patient NPO and
back and is relieved by sitting forward. Management? give IV fluids, O2, analgesia, and “tincture of time.”
--------------------------------------------------------------------------------------------------------------------------------------------
Hematology/Oncology Rapid Review – First Aid For USMLE Step 2 (3rd, Le)
Four causes of microcytic anemia. TICS—Thalassemia, Iron deficiency, anemia of
Chronic disease, and Sideroblastic anemia.
An elderly man with hypochromic, microcytic anemia Fecal occult blood test and sigmoidoscopy; suspect
is asymptomatic. Diagnostic tests? colorectal cancer.
Precipitants of hemolytic crisis in patients with G6PD Sulfonamides, antimalarial drugs, fava beans.
deficiency.
The most common inherited cause of Factor V Leiden mutation.
hypercoagulability.
The most common inherited bleeding disorder. von Willebrand’s disease.
The most common inherited hemolytic anemia. Hereditary spherocytosis.
Diagnostic test for hereditary spherocytosis. Osmotic fragility test.
Pure RBC aplasia. Diamond-Blackfan anemia.
Anemia associated with absent radii and thumbs, Fanconi’s anemia.
diffuse hyperpigmentation, café au lait spots,
microcephaly, and pancytopenia.
Medications and viruses that lead to aplastic anemia. Chloramphenicol, sulfonamides, radiation, HIV,
chemotherapeutic agents, hepatitis, parvovirus B19,
EBV.
How to distinguish polycythemia vera from 2° Both have ↑ hematocrit and RBC mass, but
polycythemia. polycythemia vera should have normal O2 saturation
and low erythropoietin levels.
Thrombotic thrombocytopenic purpura (TTP) pentad? “FAT RN”: Fever, Anemia, Thrombocytopenia, Renal
dysfunction, Neurologic abnormalities.
HUS triad? Anemia, thrombocytopenia, and acute renal failure.
Treatment for TTP. Emergent large-volume plasmapheresis,
corticosteroids, antiplatelet drugs.
Treatment for idiopathic thrombocytopenic purpura Usually resolves spontaneously; may require IVIG
(ITP) in children. and/or corticosteroids.
Which of the following are ↑ in DIC: fibrin split Fibrin split products and D-dimer are elevated;
products, D-dimer, fibrinogen, platelets, and platelets, fibrinogen, and hematocrit are ↓.
hematocrit.
An eight-year-old boy presents with hemarthrosis and ↑ Hemophilia A or B; consider desmopressin (for
PTT with normal PT and bleeding time. Diagnosis? hemophilia A) or factor VIII or IX supplements.
Treatment?
A 14-year-old girl presents with prolonged bleeding von Willebrand’s disease; treat with desmopressin,
after dental surgery and with menses, normal PT, FFP, or cryoprecipitate.
normal or ↑ PTT, and ↑ bleeding time. Diagnosis?
Treatment?
A 60-year-old African-American man presents with Monoclonal gammopathy, Bence Jones proteinuria,
bone pain. Workup for multiple myeloma might “punched-out” lesions on x-ray of the skull and long
reveal? bones.
Reed-Sternberg cells. Hodgkin’s lymphoma.
A 10-year-old boy presents with fever, weight loss, and Non-Hodgkin’s lymphoma.
night sweats. Exam shows an anterior mediastinal
mass. Suspected diagnosis?

DO NOT DISTRIBUTE -8-


Study Notes – Family Medicine & Physical ExamJames Lamberg 28Jul2010

Microcytic anemia with ↓ serum iron, ↓ total iron- Anemia of chronic disease.
binding capacity (TIBC), and normal or ↑ ferritin.
Microcytic anemia with ↓ serum iron, ↓ ferritin, and ↑ Iron deficiency anemia.
TIBC.
An 80-year-old man presents with fatigue, Chronic lymphocytic leukemia (CLL).
lymphadenopathy, splenomegaly, and isolated
lymphocytosis. Suspected diagnosis?
The lymphoma equivalent of CLL. Small lymphocytic lymphoma.
A late, life-threatening complication of chronic Blast crisis (fever, bone pain, splenomegaly,
myelogenous leukemia (CML). pancytopenia).
Auer rods on blood smear. Acute myelogenous leukemia (AML).
AML subtype associated with DIC. M3.
Electrolyte changes in tumor lysis syndrome. ↓ Ca2+, ↑ K+, ↑ phosphate, ↑ uric acid.
Treatment for AML M3. Retinoic acid.
A 50-year-old man presents with early satiety, CML.
splenomegaly, and bleeding. Cytogenetics show
t(9,22). Diagnosis?
Heinz bodies? Intracellular inclusions seen in thalassemia, G6PD
deficiency, and postsplenectomy.
An autosomal-recessive disorder with a defect in the Glanzmann’s thrombasthenia.
GPIIbIIIa platelet receptor and ↓ platelet aggregation.
Virus associated with aplastic anemia in patients with Parvovirus B19.
sickle cell anemia.
A 25-year-old African-American man with sickle cell O2, analgesia, hydration, and, if severe, transfusion.
anemia has sudden onset of bone pain. Management of
pain crisis?
A significant cause of morbidity in thalassemia Iron overload; use deferoxamine.
patients. Treatment?
--------------------------------------------------------------------------------------------------------------------------------------------
Infectious Disease Rapid Review – First Aid For USMLE Step 2 (3rd, Le)
The three most common causes of fever of unknown Infection, cancer, and autoimmune disease.
origin (FUO).
Four signs and symptoms of streptococcal pharyngitis. Fever, pharyngeal erythema, tonsillar exudate, lack of
cough.
A nonsuppurative complication of streptococcal Postinfectious glomerulonephritis.
infection that is not altered by treatment of 1° infection.
Asplenic patients are particularly susceptible to these Encapsulated organisms—pneumococcus,
organisms. meningococcus, Haemophilus influenzae, Klebsiella.
The number of bacteria on a clean-catch specimen to 10^5 bacteria/mL.
diagnose a UTI.
Which healthy population is susceptible to UTIs? Pregnant women. Treat this group aggressively because
of potential complications.
A patient from California or Arizona presents with Coccidioidomycosis. Amphotericin B.
fever, malaise, cough, and night sweats. Diagnosis?
Treatment?
Nonpainful chancre. 1° syphilis.
A “blueberry muffin” rash is characteristic of what Rubella.
congenital infection?
Meningitis in neonates. Causes? Treatment? Group B strep, E. coli, Listeria. Treat with gentamicin
and ampicillin.
Meningitis in infants. Causes? Treatment? Pneumococcus, meningococcus, H. influenzae. Treat
with cefotaxime and vancomycin.

DO NOT DISTRIBUTE -9-


Study Notes – Family Medicine & Physical ExamJames Lamberg 28Jul2010

What should always be done prior to LP? Check for ↑ ICP; look for papilledema.
CSF: Low glucose, PMN predominance Bacterial meningitis
CSF: Normal glucose, lymphocytic predominance Aseptic (viral) meningitis
CSF: Numerous RBCs in serial CSF samples Subarachnoid hemorrhage (SAH)
CSF: ↑ gamma globulins MS
Initially presents with a pruritic papule with regional Cutaneous anthrax. Treat with penicillin G or
lymphadenopathy; evolves into a black eschar after 7– ciprofloxacin.
10 days. Treatment?
Findings in 3° syphilis. Tabes dorsalis, general paresis, gummas, Argyll
Robertson pupil, aortitis, aortic root aneurysms.
Characteristics of 2° Lyme disease. Arthralgias, migratory polyarthropathies, Bell’s palsy,
myocarditis.
Cold agglutinins. Mycoplasma.
A 24-year-old man presents with soft white plaques on Candidal thrush. Workup should include an HIV test.
his tongue and the back of his throat. Diagnosis? Treat with nystatin oral suspension.
Workup? Treatment?
Begin Pneumocystis jiroveci (formerly P. carinii) ≤ 200 for P. jiroveci (with TMP-SMX); ≤ 50–100 for
pneumonia prophylaxis in an HIV-positive patient at MAI (with clarithromycin/azithromycin).
what CD4 count? Mycobacterium avium–intracellulare
(MAI) prophylaxis?
Risk factors for pyelonephritis. Pregnancy, vesicoureteral reflux, anatomic anomalies,
indwelling catheters, kidney stones.
Neutropenic nadir postchemotherapy. 7–10 days.
Erythema migrans. Lesion of 1° Lyme disease.
Classic physical findings for endocarditis. Fever, heart murmur, Osler’s nodes, splinter
hemorrhages, Janeway lesions, Roth’s spots.
Aplastic crisis in sickle cell disease. Parvovirus B19.
Ring-enhancing brain lesion on CT with seizures. Taenia solium (cysticercosis).
Organism: Branching rods in oral infection Actinomyces israelii
Organism: Painful chancroid Haemophilus ducreyi
Organism: Dog or cat bite Pasteurella multocida
Organism: Gardener Sporothrix schenckii
Organism: Pregnant women with pets Toxoplasma gondii
Organism: Meningitis in adults Neisseria meningitidis
Organism: Meningitis in elderly Streptococcus pneumoniae
Organism: Alcoholic with pneumonia Klebsiella
Organism: "Currant jelly" sputum Klebsiella
Organism: Infection in burn victims Pseudomonas
Organism: Osteomyelitis from foot wound puncture Pseudomonas
Organism: Osteomyelitis in sickle cell patient Salmonella
A 55-year-old man who is a smoker and a heavy Legionella pneumonia.
drinker presents with a new cough and flulike
symptoms. Gram stain shows no organisms; silver stain
of sputum shows gram-negative rods. What is the
diagnosis?
A middle-aged man presents with acute-onset Lyme disease, Ixodes tick, doxycycline.
monoarticular joint pain and bilateral Bell’s palsy.
What is the likely diagnosis, and how did he get it?
Treatment?
A patient develops endocarditis three weeks after S. aureus or S. epidermidis.
receiving a prosthetic heart valve. What organism is
suspected?

DO NOT DISTRIBUTE - 10 -
Study Notes – Family Medicine & Physical ExamJames Lamberg 28Jul2010

--------------------------------------------------------------------------------------------------------------------------------------------
Musculoskeletal Rapid Review – First Aid For USMLE Step 2 (3rd, Le)
Back pain that is exacerbated by standing and walking Spinal stenosis.
and relieved with sitting and hyperflexion of the hips.
Joints in the hand affected in rheumatoid arthritis. MCP and PIP joints; DIP joints are spared.
Joint pain and stiffness that worsen over the course of Osteoarthritis.
the day and are relieved by rest.
Genetic disorder associated with multiple fractures and Osteogenesis imperfecta.
commonly mistaken for child abuse.
Hip and back pain along with stiffness that improves Suspect ankylosing spondylitis. Check HLA-B27.
with activity over the course of the day and worsens at
rest. Diagnostic test?
Arthritis, conjunctivitis, and urethritis in young men. Reactive (Reiter’s) arthritis. Associated with
Associated organisms? Campylobacter, Shigella, Salmonella, Chlamydia, and
Ureaplasma.
A 55-year-old man has sudden, excruciating first MTP Gout. Needle-shaped, negatively birefringent crystals
joint pain after a night of drinking red wine. Diagnosis, are seen on joint fluid aspirate. Chronic treatment with
workup, and chronic treatment? allopurinol or probenecid.
Rhomboid-shaped, positively birefringent crystals on Pseudogout.
joint fluid aspirate.
An elderly woman presents with pain and stiffness of Polymyalgia rheumatica.
the shoulders and hips; she cannot lift her arms above
her head. Labs show anemia and ↑ ESR.
An active 13-year-old boy has anterior knee pain. Osgood-Schlatter disease.
Diagnosis?
Bone is fractured in a fall on an outstretched hand. Distal radius (Colles’ fracture).
Complication of scaphoid fracture. Avascular necrosis.
Signs suggesting radial nerve damage with humeral Wrist drop, loss of thumb abduction.
fracture.
A young child presents with proximal muscle Duchenne muscular dystrophy.
weakness, waddling gait, and pronounced calf muscles.
A first-born female who was born in breech position is Developmental dysplasia of the hip. If severe, consider
found to have asymmetric skin folds on her newborn a Pavlik harness to maintain abduction.
exam. Diagnosis? Treatment?
An 11-year-old obese African-American boy presents Slipped capital femoral epiphysis. AP and frog-leg
with sudden onset of limp. Diagnosis? Workup? lateral view.
The most common 1° malignant tumor of bone. Multiple myeloma.
--------------------------------------------------------------------------------------------------------------------------------------------
Neurology Rapid Review – First Aid For USMLE Step 2 (3rd, Le)
Unilateral, severe periorbital headache with tearing and Cluster headache.
conjunctival erythema.
Prophylactic treatment for migraine. Antihypertensives, antidepressants, anticonvulsants.
The most common pituitary tumor. Treatment? Prolactinoma. Dopamine agonists (e.g., bromocriptine).
A 55-year-old patient presents with acute “broken Broca’s aphasia. Frontal lobe, left MCA distribution.
speech.” What type of aphasia? What lobe and vascular
distribution?
The most common cause of SAH. Trauma; the second most common is berry aneurysm.
A crescent-shaped hyperdensity on CT that does not Subdural hematoma—bridging veins torn.
cross the midline.
A history significant for initial altered mental status Epidural hematoma. Middle meningeal artery.
with an intervening lucid interval. Diagnosis? Most Neurosurgical evacuation.
likely source? Treatment?
CSF findings with SAH. Elevated ICP, RBCs, xanthochromia.

DO NOT DISTRIBUTE - 11 -
Study Notes – Family Medicine & Physical ExamJames Lamberg 28Jul2010

Albuminocytologic dissociation. Guillain-Barré syndrome (↑ protein in CSF without a


significant ↑ in cell count).
Cold water is flushed into a patient’s ear, and the fast Normal.
phase of the nystagmus is toward the opposite side.
Normal or pathologic?
The most common 1° sources of metastases to the Lung, breast, skin (melanoma), kidney, GI tract.
brain.
May be seen in children who are accused of inattention Absence seizures.
in class and confused with ADHD.
The most frequent presentation of intracranial Headache.
neoplasm.
The most common cause of seizures in children (2–10 Infection, febrile seizures, trauma, idiopathic.
years).
The most common cause of seizures in young adults Trauma, alcohol withdrawal, brain tumor.
(18–35 years).
First-line medication for status epilepticus. IV benzodiazepine.
Confusion, confabulation, ophthalmoplegia, ataxia. Wernicke’s encephalopathy due to a deficiency of
thiamine.
What % lesion is an indication for carotid Seventy percent if the stenosis is symptomatic.
endarterectomy?
The most common causes of dementia. Alzheimer’s and multi-infarct.
Combined UMN and LMN disorder. ALS.
Rigidity and stiffness with resting tremor and masked Parkinson’s disease.
facies.
The mainstay of Parkinson’s therapy. Levodopa/carbidopa.
Treatment for Guillain-Barré syndrome. IVIG or plasmapheresis.
Rigidity and stiffness that progress to choreiform Huntington’s disease.
movements, accompanied by moodiness and altered
behavior.
A six-year-old girl presents with a port-wine stain in Sturge-Weber syndrome. Treat symptomatically.
the V2 distribution as well as with mental retardation, Possible focal cerebral resection of the affected lobe.
seizures, and ipsilateral leptomeningeal angioma.
Café au lait spots on skin. Neurofibromatosis type 1.
Hyperphagia, hypersexuality, hyperorality, and Klüver-Bucy syndrome (amygdala).
hyperdocility.
May be administered to a symptomatic patient to Edrophonium.
diagnose myasthenia gravis.
--------------------------------------------------------------------------------------------------------------------------------------------
Obstetrics Rapid Review – First Aid For USMLE Step 2 (3rd, Le)
1° causes of third-trimester bleeding. Placental abruption and placenta previa.
Classic ultrasound and gross appearance of complete Snowstorm on ultrasound. “Cluster-of-grapes”
hydatidiform mole. appearance on gross examination.
Chromosomal pattern of a complete mole. 46,XX.
Molar pregnancy containing fetal tissue. Partial mole.
Symptoms of placental abruption. Continuous, painful vaginal bleeding.
Symptoms of placenta previa. Self-limited, painless vaginal bleeding.
When should a vaginal exam be performed with Never.
suspected placenta previa?
Antibiotics with teratogenic effects. Tetracycline, fluoroquinolones, aminoglycosides,
sulfonamides.
Shortest AP diameter of the pelvis. Obstetric conjugate: between the sacral promontory
and the midpoint of the symphysis pubis.

DO NOT DISTRIBUTE - 12 -
Study Notes – Family Medicine & Physical ExamJames Lamberg 28Jul2010

Medication given to accelerate fetal lung maturity. Betamethasone or dexamethasone × 48 hours.


The most common cause of postpartum hemorrhage. Uterine atony.
Treatment for postpartum hemorrhage. Uterine massage; if that fails, give oxytocin.
Typical antibiotics for group B streptococcus (GBS) IV penicillin or ampicillin.
prophylaxis.
A patient fails to lactate after an emergency C-section Sheehan’s syndrome (postpartum pituitary necrosis).
with marked blood loss.
Uterine bleeding at 18 weeks’ gestation; no products Inevitable abortion.
expelled; membranes ruptured; cervical os open.
Uterine bleeding at 18 weeks’ gestation; no products Threatened abortion.
expelled; cervical os closed.
--------------------------------------------------------------------------------------------------------------------------------------------
Gynecology Rapid Review – First Aid For USMLE Step 2 (3rd, Le)
The first test to perform when a woman presents with β-hCG; the most common cause of amenorrhea is
amenorrhea. pregnancy.
Term for heavy bleeding during and between menstrual Menometrorrhagia.
periods.
Cause of amenorrhea with normal prolactin, no Asherman’s syndrome.
response to estrogen-progesterone challenge, and a
history of D&C.
Therapy for polycystic ovarian syndrome. Weight loss and OCPs.
Medication used to induce ovulation. Clomiphene citrate.
Diagnostic step required in a postmenopausal woman Endometrial biopsy.
who presents with vaginal bleeding.
Indications for medical treatment of ectopic pregnancy. Stable, unruptured ectopic pregnancy of < 3.5 cm at <
6 weeks’ gestation.
Medical options for endometriosis. OCPs, danazol, GnRH agonists.
Laparoscopic findings in endometriosis. “Chocolate cysts,” powder burns.
The most common location for an ectopic pregnancy. Ampulla of the oviduct.
How to diagnose and follow a leiomyoma. Ultrasound.
Natural history of a leiomyoma. Regresses after menopause.
A patient has ↑ vaginal discharge and petechial patches Trichomonas vaginitis.
in the upper vagina and cervix.
Treatment for bacterial vaginosis. Oral or topical metronidazole.
The most common cause of bloody nipple discharge. Intraductal papilloma.
Contraceptive methods that protect against PID. OCPs and barrier contraception.
Unopposed estrogen is contraindicated in which Endometrial or estrogen receptor positive breast
cancers? cancer.
A patient presents with recent PID with RUQ pain. Consider Fitz-Hugh-Curtis syndrome.
Breast malignancy presenting as itching, burning, and Paget’s disease.
erosion of the nipple.
Annual screening for women with a strong family CA-125 and transvaginal ultrasound.
history of ovarian cancer.
A 50-year-old woman leaks urine when laughing or Kegel exercises, estrogen, pessaries for stress
coughing. Nonsurgical options? incontinence.
A 30-year-old woman has unpredictable urine loss. Anticholinergics (oxybutynin) or β-adrenergics
Examination is normal. Medical options? (metaproterenol) for urge incontinence.
Lab values suggestive of menopause. ↑ serum FSH.
The most common cause of female infertility. Endometriosis.
Two consecutive findings of atypical squamous cells of Colposcopy and endocervical curettage.
undetermined significance (ASCUS) on Pap smear.
Follow-up evaluation?

DO NOT DISTRIBUTE - 13 -
Study Notes – Family Medicine & Physical ExamJames Lamberg 28Jul2010

Breast cancer type that ↑ the future risk of invasive Lobular carcinoma in situ.
carcinoma in both breasts.
--------------------------------------------------------------------------------------------------------------------------------------------
Pediatrics Rapid Review – First Aid For USMLE Step 2 (3rd, Le)
Nontender abdominal mass associated with elevated Neuroblastoma.
VMA and HVA.
The most common type of tracheoesophageal fistula Esophageal atresia with distal TEF (85%). Unable to
(TEF). Diagnosis? pass NG tube.
Not contraindications to vaccination. Mild illness and/or low-grade fever, current antibiotic
therapy, and prematurity.
Tests to rule out shaken baby syndrome. Ophthalmologic exam, CT, and MRI.
A neonate has meconium ileus. CF or Hirschsprung’s disease.
Bilious emesis within hours after the first feeding. Duodenal atresia.
A two-month-old baby presents with nonbilious Correct metabolic abnormalities. Then correct pyloric
projectile emesis. What are the appropriate steps in stenosis with pyloromyotomy.
management?
The most common 1° immunodeficiency. Selective IgA deficiency.
An infant has a high fever and onset of rash as fever Febrile seizures (roseola infantum).
breaks. What is he at risk for?
Immunodeficiency: A boy has chronic respiratory Chronic granulomatous disease
infections. Nitroblue tetrazolium test is positive.
Immunodeficiency: A child has eczema, Wiskott-Aldrich syndrome
thrombocytopenia, and high levels of IgA.
Immunodeficiency: A four-month-old boy has life- Bruton’s X-linked agammaglobulinemia
threatening Pseudomonas infection.
Acute-phase treatment for Kawasaki disease. High-dose aspirin for inflammation and fever; IVIG to
prevent coronary artery aneurysms.
Treatment for mild and severe unconjugated Phototherapy (mild) or exchange transfusion (severe).
hyperbilirubinemia.
Sudden onset of mental status changes, emesis, and Reye’s syndrome.
liver dysfunction after taking aspirin.
A child has loss of red light reflex. Diagnosis? Suspect retinoblastoma.
Vaccinations at a six-month well-child visit. HBV, DTaP, Hib, IPV, PCV.
Tanner stage 3 in a six-year-old girl. Precocious puberty.
Infection of small airways with epidemics in winter RSV bronchiolitis.
and spring.
Cause of neonatal RDS. Surfactant deficiency.
A condition associated with red “currant-jelly” stools. Intussusception.
A congenital heart disease that causes 2° hypertension. Coarctation of the aorta.
First-line treatment for otitis media. Amoxicillin × 10 days.
The most common pathogen causing croup. Parainfluenza virus type 1.
A homeless child is small for his age and has peeling Kwashiorkor (protein malnutrition).
skin and a swollen belly.
Defect in an X-linked syndrome with mental Lesch-Nyhan syndrome (purine salvage problem with
retardation, gout, self-mutilation, and choreoathetosis. HGPRTase deficiency).
A newborn girl has a continuous “machinery murmur.” Patent ductus arteriosus (PDA).
--------------------------------------------------------------------------------------------------------------------------------------------
Psychiatry Rapid Review – First Aid For USMLE Step 2 (3rd, Le)
First-line pharmacotherapy for depression. SSRIs.
Antidepressants associated with hypertensive crisis. MAOIs.
Galactorrhea, impotence, menstrual dysfunction, and ↓ Patient on dopamine antagonist.
libido.

DO NOT DISTRIBUTE - 14 -
Study Notes – Family Medicine & Physical ExamJames Lamberg 28Jul2010

A 17-year-old girl has left arm paralysis after her Conversion disorder.
boyfriend dies in a car crash. No medical cause found.
Defense mechanism: A mother who is angry at her Displacement
husband yells at her child.
Defense mechanism: A pedophile enters a monastery. Reaction formation
Defense mechanism: A woman calmly describes a Isolation
grisly murder.
Defense mechanism: A hospitalized 10-year-old begins Regression
to wet his bed.
Life-threatening muscle rigidity, fever, and Neuroleptic malignant syndrome.
rhabdomyolysis.
Amenorrhea, bradycardia, and abnormal body image in Anorexia.
a young female.
A 35-year-old man has recurrent episodes of Panic disorder.
palpitations, diaphoresis, and fear of going crazy.
The most serious side effect of clozapine. Agranulocytosis.
A 21-year-old man has three months of social Schizophreniform disorder (diagnosis of schizophrenia
withdrawal, worsening grades, flattened affect, and requires ≥ 6 months of symptoms).
concrete thinking.
Key side effects of atypical antipsychotics. Weight gain, type 2 DM, QT prolongation.
A young weight lifter receives IV haloperidol and Acute dystonia (oculogyric crisis). Treat with
complains that his eyes are deviated sideways. benztropine or diphenhydramine.
Diagnosis? Treatment?
Medication to avoid in patients with a history of Neuroleptics.
alcohol withdrawal seizures.
A 13-year-old boy has a history of theft, vandalism, Conduct disorder.
and violence toward family pets.
A five-month-old girl has ↓ head growth, truncal Rett’s disorder.
dyscoordination, and ↓ social interaction.
A patient hasn’t slept for days, lost $20,000 gambling, Acute mania. Start a mood stabilizer (e.g., lithium).
is agitated, and has pressured speech. Diagnosis?
Treatment?
After a minor fender bender, a man wears a neck brace Malingering.
and requests permanent disability.
A nurse presents with severe hypoglycemia; blood Factitious disorder (Munchausen syndrome).
analysis reveals no elevation in C-peptide.
A patient continues to use cocaine after being in jail, Substance abuse.
losing his job, and not paying child support.
A violent patient has vertical/horizontal nystagmus. Phencyclidine hydrochloride (PCP) intoxication.
A woman who was abused as a child frequently feels Depersonalization disorder.
outside of or detached from her body.
A man has repeated, intense urges to rub his body Frotteurism (a paraphilia).
against unsuspecting passengers on a bus.
A schizophrenic patient takes haloperidol for one year Tardive dyskinesia. ↓ or discontinue haloperidol and
and develops uncontrollable tongue movements. consider another antipsychotic (e.g., risperidone,
Diagnosis? Treatment? clozapine).
A man unexpectedly flies across the country, takes a Dissociative fugue.
new name, and has no memory of his prior life.
--------------------------------------------------------------------------------------------------------------------------------------------
Pulmonary Rapid Review – First Aid For USMLE Step 2 (3rd, Le)
Risk factors for DVT. Stasis, endothelial injury, and hypercoagulability
(Virchow’s triad).

DO NOT DISTRIBUTE - 15 -
Study Notes – Family Medicine & Physical ExamJames Lamberg 28Jul2010

Criteria for exudative effusion. Pleural/serum protein > 0.5; pleural/serum LDH > 0.6.
Causes of exudative effusion. Think of leaky capillaries. Malignancy, TB, bacterial
or viral infection, pulmonary embolism with infarct,
and pancreatitis.
Causes of transudative effusion. Think of intact capillaries. CHF, liver or kidney
disease, and protein-losing enteropathy.
Normalizing PCO2 in a patient having an asthma Fatigue and impending respiratory failure.
exacerbation may indicate?
Dyspnea, bilateral hilar lymphadenopathy on CXR, Sarcoidosis.
noncaseating granulomas, ↑ ACE, and hypercalcemia.
PFTs showing ↓ FEV1/FVC. Obstructive pulmonary disease (e.g., asthma).
PFTs showing ↑ FEV1/FVC. Restrictive pulmonary disease.
Honeycomb pattern on CXR. Diagnosis? Treatment? Diffuse interstitial pulmonary fibrosis. Supportive care.
Steroids may help.
Treatment for SVC syndrome. Radiation.
Treatment for mild, persistent asthma. Inhaled β-agonists and inhaled corticosteroids.
Treatment for COPD exacerbation. O2, bronchodilators, antibiotics, corticosteroids with
taper, smoking cessation.
Treatment for chronic COPD. Smoking cessation, home O2, β-agonists,
anticholinergics, systemic or inhaled corticosteroids,
flu and pneumococcal vaccines.
Acid-base disorder in pulmonary embolism. Hypoxia and hypocarbia (respiratory alkalosis).
Non–small cell lung cancer (NSCLC) associated with Squamous cell carcinoma.
hypercalcemia.
Lung cancer associated with SIADH. Small cell lung cancer (SCLC).
Lung cancer highly related to cigarette exposure. SCLC.
A tall white male presents with acute shortness of Spontaneous pneumothorax. Spontaneous regression.
breath. Diagnosis? Treatment? Supplemental O2 may be helpful.
Treatment of tension pneumothorax. Immediate needle thoracostomy.
Characteristics favoring carcinoma in an isolated Age > 45–50 years; lesions new or larger in
pulmonary nodule. comparison to old films; absence of calcification or
irregular calcification; size > 2 cm; irregular margins.
Hypoxemia and pulmonary edema with normal ARDS.
pulmonary capillary wedge pressure.
Sequelae of asbestos exposure. Pulmonary fibrosis, pleural plaques, bronchogenic
carcinoma (mass in lung field), mesothelioma (pleural
mass).
↑ risk of what infection with silicosis? Mycobacterium tuberculosis.
Causes of hypoxemia. Right-to-left shunt, hypoventilation, low inspired O2
tension, diffusion defect, V/Q mismatch.
Classic CXR findings for pulmonary edema. Cardiomegaly, prominent pulmonary vessels, Kerley B
lines, “bat’s-wing” appearance of hilar shadows, and
perivascular and peribronchial cuffing.
--------------------------------------------------------------------------------------------------------------------------------------------
Renal/Genitourinary Rapid Review – First Aid For USMLE Step 2 (3rd, Le)
Renal tubular acidosis (RTA) associated with abnormal Type I (distal) RTA.
H+ secretion and nephrolithiasis.
RTA associated with abnormal HCO3– and rickets. Type II (proximal) RTA.
RTA associated with aldosterone defect. Type IV (distal) RTA.
“Doughy” skin Hypernatremia.
Differential of hypervolemic hyponatremia. Cirrhosis, CHF, nephritic syndrome.
Chvostek’s and Trousseau’s signs. Hypocalcemia.

DO NOT DISTRIBUTE - 16 -
Study Notes – Family Medicine & Physical ExamJames Lamberg 28Jul2010

The most common causes of hypercalcemia. Malignancy and hyperparathyroidism.


T-wave flattening and U waves. Hypokalemia.
Peaked T waves and widened QRS. Hyperkalemia.
First-line treatment for moderate hypercalcemia. IV hydration and loop diuretics (furosemide).
Type of ARF in a patient with FeNa < 1%. Prerenal.
A 49-year-old man presents with acute-onset flank pain Nephrolithiasis.
and hematuria.
The most common type of nephrolithiasis. Calcium oxalate.
A 20-year-old man presents with a palpable flank mass Cerebral berry aneurysms (autosomal-dominant
and hematuria. Ultrasound shows bilateral enlarged PCKD).
kidneys with cysts. Associated brain anomaly?
Hematuria, hypertension, and oliguria. Nephritic syndrome.
Proteinuria, hypoalbuminemia, hyperlipidemia, Nephrotic syndrome.
hyperlipiduria, and edema.
The most common form of nephritic syndrome. Nephrotic syndrome.
The most common form of glomerulonephritis. IgA nephropathy (Berger’s disease).
Glomerulonephritis with deafness. Alport’s syndrome.
Glomerulonephritis with hemoptysis. Wegener’s granulomatosis and Goodpasture’s
syndrome.
Presence of red cell casts in urine sediment. Glomerulonephritis/nephritic syndrome.
Eosinophils in urine sediment. Allergic interstitial nephritis.
Waxy casts in urine sediment and Maltese crosses Nephrotic syndrome.
(seen with lipiduria).
Drowsiness, asterixis, nausea, and a pericardial friction Uremic syndrome seen in patients with renal failure.
rub.
A 55-year-old man is diagnosed with prostate cancer. Wait, surgical resection, radiation and/or androgen
Treatment options? suppression.
Low urine specific gravity in the presence of high Diabetes insipidus.
serum osmolality.
Treatment of SIADH? Fluid restriction, demeclocycline.
Hematuria, flank pain, and palpable flank mass. Renal cell carcinoma (RCC).
Testicular cancer associated with β-hCG, AFP. Choriocarcinoma.
The most common type of testicular cancer. Seminoma, a type of germ cell tumor.
The most common histology of bladder cancer. Transitional cell carcinoma.
Complication of overly rapid correction of Central pontine myelinolysis.
hyponatremia.
Salicylate ingestion occurs in what type of acid-base Anion gap acidosis and 1° respiratory alkalosis due to
disorder? central respiratory stimulation.
Acid-base disturbance commonly seen in pregnant Respiratory alkalosis.
women.
Three systemic diseases that lead to nephrotic DM, SLE, and amyloidosis.
syndrome.
Elevated erythropoietin level, elevated hematocrit, and RCC or other erythropoietin-producing tumor; evaluate
normal O2 saturation suggest? with CT scan.
A 55-year-old male presents with irritative and Likely BPH. Options include no treatment, terazosin,
obstructive urinary symptoms. Treatment options? finasteride, or surgical intervention (TURP).
--------------------------------------------------------------------------------------------------------------------------------------------
Top 100 Secrets – Physical Diagnosis Secrets (2nd, Mangione)
1) A postural dizziness (severe enough to stop the test) or an increase in heart rate of at least 30 beats/minute has
sensitivity of 97% and specificity of 96% for blood loss >630mL. Unless associated with dizziness, postural
hypotension of any degree has little value.

DO NOT DISTRIBUTE - 17 -
Study Notes – Family Medicine & Physical ExamJames Lamberg 28Jul2010

2) Body fat "distributions" by waist circumference (WC) and waist-to-hip ratio (WHR) are much better markers for
cardiovascular risk than the body mass index (BMI) alone. In fact, a WC <100 cm practically excludes insulin
resistance.
3) An acute difference in systolic pressure >20 mmHg between the two arms usually indicates aortic dissection
(complicated by aortic regurgitation in cases of more proximal dissection). If chronic, it indicates instead a
subclavian artery occlusion or a subclavian steal syndrome.
4) An ankle-to-arm systolic pressure index (AAI) <0.97 identifies patients with angiographically proven
occlusions/stenoses of lower extremities arteries with 96% sensitivity and 94-100% specificity. Most patients with
claudication will have AAI values between 0.5 and 0.8, whereas those with pain at rest will have values <0.5.
Indexes <0.2 are associated with ischemic or gangrenous extremities.
5) Paired, transverse, white nail bands in the second, third, and fourth fingers (Muehrcke lines) suggest chronic
hypoalbuminemia, occurring in more than three quarters of patients with nephrotic syndrome (<2.3 gm/100mL) but
also in liver disease and malnutrition.
6) In a study of 118 subjects with acrochordons (skin tags), 41% had either impaired glucose tolerance or overt type
2 diabetes.
7) Ten percent of patients with vitiligo have serologic or clinical evidence of autoimmune disorders; the most
common are thyroid diseases, especially hypothyroidism of the Hashimoto variety. Diabetes, Addison's, pernicious
anemia, alopecia areata, and uveitis (Vogt-Koyanagi syndrome) also are frequent.
8) Twenty percent of patients with acanthosis nigricans (AN) have an aggressive underlying neoplasm-a
gastrointestinal (GI) adenocarcinoma in 90% of cases, and a gastric in 60%. Still, most patients with AN have just
obesity and insulin resistance.
9) To separate icterus from the brownish color normally present in the bulbar conjunctiva of dark-skinned
individuals, ask the patient to look upward. Then inspect the inferior conjunctival recess. This should be entirely
white in nonicteric subjects, since the brownish discoloration of these individuals is the result of sunlight exposure.
10) Earliest signs of nonproliferative diabetic retinopathy include microaneurysms and dot intraretinal hemorrhages,
with progression of disease characterized by an increase in number and size of microaneurysms and intraretinal
hemorrhages (both dot and blot). Soft exudates are not as predictive, and hard exudates even less.
11) Diagonal earlobe creases in adults are an acquired phenomenon and a significant independent variable for
coronary artery disease. Hair in the external ear canal also seems to be associated with coronary artery disease.
12) Findings that can best separate patients with and without strep throat are (1) pharyngeal or tonsillar exudates, (2)
fever by history, (3) tonsillar enlargement, (4) tenderness or enlargement of the anterior cervical and jugulodigastric
lymph nodes, and (5) absence of cough.
13) Multiple white, warty, corrugated, and painless plaques on the lateral margins of the tongue (hairy leukoplakia)
represent an Epstein-Barr-induced lesion typical of HIV infection, even though this can also occur in severely
immunocompromised organ transplant patients. If present, it carries a worse prognosis for HIV progression.
14) Pemberton's maneuver (reversible superior vena cava obstruction caused by a substernal goiter being "lifted"
into the thoracic inlet as a result of arm raising) is a nonspecific finding that may be encountered in patients with
substernal thyroid masses, lymphomas, or upper mediastinal tumors.
15) The average size of a thyroid nodule detected on exam is 3 cm. In fact, the larger the nodule, the more likely its
detection (with <1 cm nodules being missed 90% of the time; <2 cm nodules 50% of the time).
16) Findings most suggestive of hyperthyroidism include lid retraction (likelihood ratio [LR] = 31.5), lid lag (LR =
17.6), fine finger tremor (LR = 11.4), moist and warm skin (LR = 6.7), and tachycardia (LR = 4.4). Findings more
likely to rule out hyperthyroidism are normal thyroid size (LR = 0.1), heart rate <90/minute (LR = 0.2), and no
finger tremor (LR = 0.3). Older hyperthyroid patients exhibit more anorexia and atrial fibrillation; more frequent
lack of goiter; and overall fewer signs, with tachycardia, fatigue, and weight loss in more than 50% of patients (and
all three in 32%).
17) Findings more strongly suggestive of hypothyroidism are bradycardia (LR = 3.88), abnormal ankle reflex (LR =
3.41), and coarse skin (LR = 2.3). No single finding, when absent, can effectively rule out hypothyroidism.
18) Clinical breast exam (CBE) has low sensitivity for the detection of breast masses, high specificity, and accuracy
that can be increased by (1) longer duration of exam (at least 3 minutes per breast); (2) higher number of correct
steps (a systematic and vertical search pattern, thoroughness, varying palpation pressure, use of three fingers, finger
pads, and circular motion); and (3) examiner experience (previous training with silicone models).
19) A brisk arterial upstroke with a widened pulse pressure indicates aortic regurgitation (AR). A brisk arterial
upstroke with a normal pulse pressure instead indicates either the simultaneous emptying of the left ventricle into a
high pressure bed (the aorta) and a lower pressure bed (like the right ventricle in patients with ventricular septal
defect, or the left atrium in patients with mitral regurgitation) or hypertrophic obstructive cardiomyopathy (HOCM).

DO NOT DISTRIBUTE - 18 -
Study Notes – Family Medicine & Physical ExamJames Lamberg 28Jul2010

20) The alternation of strong and weak arterial pulses despite regular rate and rhythm (pulsus alternans) indicates
severe left ventricular dysfunction, with worse ejection fraction and higher pulmonary capillary pressure. Hence, it
is often associated with an S3 gallop.
21) Visible neck veins in the upright position indicate a central venous pressure >7 cmH2O and thus are pathologic.
22) In chronic heart failure, jugular venous distention represents an ominous prognostic variable, independently
associated with adverse outcomes, including risk of death or hospitalization. The presence of S3 is similarly (and
independently) associated with increased risk.
23) Presence of either end-inspiratory crackles or distended neck veins has high specificity (90-100%) but low
sensitivity (10-50%) for increased left-sided filling pressure due to either systolic or diastolic dysfunction.
24) Positive abdominojugular reflux has equally high specificity (but better sensitivity, 55-85%) for increased left-
sided filling pressure. S3 gallop, downward and lateral displacement of the apical impulse, and peripheral edema
also have high specificity (>95%) but low sensitivity (10-40%). Of these, only the S3 and the displaced apical
impulse have a positive likelihood ratio (5.7 and 5.8, respectively).
25) In patients presenting with dyspnea, an abdominojugular reflux argues in favor of bi-ventricular failure and
suggests a pulmonary capillary wedge pressure >15 mmHg. Conversely, a negative abdominojugular reflux in a
patient with dyspnea argues strongly against increased left atrial pressure.
26) Posturally induced crackles (PICs) after myocardial infarction (MI) carry an ominous significance, reflecting
higher pulmonary capillary wedge pressure, lower pulmonary venous compliance, and higher mortality. After the
number of diseased coronary vessels and the patient's pulmonary capillary wedge pressure, PICs rank third as most
important predictor of recovery after an acute MI.
27) Ischemic heart disease patients with S3 have a 1-year mortality that is much higher than those without it (57%
versus 14%). The same applies to a displaced apical impulse (39% versus 12%).
28) Leg swelling without increased central venous pressure (CVP) suggests bilateral venous insufficiency or
noncardiac edema (hepatic or renal).
29) The Valsalva maneuver has excellent specificity and sensitivity (90-99% and 70-95%, respectively) for
detecting left ventricular dysfunction, either systolic or diastolic.
30) The PPP (proportional pulse pressure-arterial pulse pressure divided by the systolic blood pressure) has excellent
sensitivity (91%) and specificity (83%) for identifying low cardiac index (CI). A PPP <0.25 has a positive likelihood
ratio of 5.4 for CI of 2.2 L/min/m2.
31) Patients with distended neck veins, dyspnea/tachypnea, tachycardia, and clear lungs should be thought of as
having tamponade; thus, their pulsus paradoxus must be measured.
32) A pulsus paradoxus >21 mmHg has good sensitivity and excellent specificity for tamponade. It also may be
palpable.
33) A paradoxical increase in venous distention during inspiration (Kussmaul's sign) is not a feature of tamponade
but does occur in 30-50% of patients with "pure" constrictive pericarditis; 90% of patients with constrictive
pericarditis also have a retracting apical impulse.
34) A loud S1 should always alert the clinician to the possibility of mitral stenosis and should thus prompt a search
for its associated diastolic rumble.
35) An audible physiologic splitting of S2 is age dependent, present in 60% of subjects younger than 30 and 30% of
those older than 60.
36) Wide splitting of S2 usually reflects a delayed closure of the pulmonic valve because of either a right bundle
branch block or pulmonary hypertension.
37) S2 that remains audibly split throughout respiration, both in the supine and upright positions, with a consistent
interval between its two components, argues in favor of an atrial septal defect.
38) S2 that becomes audibly split only in exhalation, while remaining single in inspiration (paradoxical or reversed
splitting), means pathology until proven otherwise. This is usually an increased impedance to left ventricular
emptying (aortic stenosis, coarctation, or hypertension), a left bundle branch block, or a transient, left ventricular
ischemia.
39) A loud and ringing S2, rich in overtones and tambour like ("drum" in French), indicates a dilation of the aortic
root. When associated with an aortic regurgitation murmur, it suggests Marfan syndrome, syphilis (Potain's sign), or
a dissecting aneurysm of the ascending aorta (Harvey's sign).
40) S3 is such an accurate predictor of systolic dysfunction (and elevated atrial pressure) that its absence argues in
favor of an ejection fraction >30%.
41) In patients with congestive heart failure, S3 is the best predictor for response to digitalis and overall mortality. It
correlates with high levels of B-type natriuretic peptide (BNP), and if associated with elevated jugular venous
pressure, it predicts more frequent hospitalizations and worse outcome. S3 is also the most significant predictor of

DO NOT DISTRIBUTE - 19 -
Study Notes – Family Medicine & Physical ExamJames Lamberg 28Jul2010

cardiac risk during noncardiac surgery. If preoperative diuresis is not instituted, it can also predict mortality. Finally,
the presence of S3 in mitral regurgitation reflects worse disease (i.e., higher filling pressure, lower ejection fraction,
and more severe regurgitation).
42) S4 reflects an increase in late ventricular diastolic pressure (hence a diastolic dysfunction); but, in contrast to S3,
it reflects normal atrial pressure, normal cardiac output, and normal ventricular diameter.
43) S4 can be heard in as many as 90% of patients with MI, but eventually resolves. Presence of S4 at more than 1
month after MI does predict a higher 5-year mortality rate.
44) An early systolic (ejection) sound indicates normal ejection of blood through an abnormal aortic valve (i.e.,
bicuspid), normal ejection of blood into a stiffened and dilated aortic root (i.e., hypertension, atherosclerosis, aortic
aneurysm, or aortic regurgitation), or forceful ejection of blood into a normal aortic root (high output states like
aortic regurgitation).
45) An aortic ES in patients with aortic regurgitation (AR) argues in favor of valvular AR, possibly due to a bicuspid
valve.
46) In mitral valve prolapse (MVP), clickers stay clickers and murmurs stay murmurs. This may have implication
for prophylaxis.
47) One tenth of all rubs are associated with a pericardial effusion. In fact, rubs can occur in up to one fourth of
tamponade cases. Hence, measure pulsus paradoxus in all patients with a rub.
48) All right-sided auscultatory findings (except the pulmonic ejection sound) get louder on inspiration (Rivero
Carvallo maneuver).
49) A murmur that intensifies with Valsalva or squatting-to-standing is due to either HOCM or MVP.
50) A longer diastolic pause (such as that following a premature beat) intensifies the murmur of aortic stenosis but
not that of mitral regurgitation.
51) A benign "functional" murmur should be systolic, short, soft (typically <3/6), early peaking (never passing
midsystole), predominantly circumscribed to the base, and associated with a well-preserved and normally split
second sound. It should have an otherwise normal cardiovascular exam (i.e., no bad company); and it often
disappears with sitting, standing, or straining (as, for example, following a Valsalva maneuver).
52) A "bad" systolic murmur instead should be long, loud (in fact, pathologic by definition if loud enough to
generate a thrill), late peaking, nonlocalized, and associated with a soft-to-absent S2 that does not normally split. It
also should be accompanied by other abnormal findings/symptoms ("bad" company).
53) The murmur of aortic sclerosis is the most common systolic ejection murmur of the elderly, affecting 21-26% of
persons older than 65 and 55-75% of octogenarians and carrying a 40% increased risk of myocardial infarction.
54) Presence of an early systolic (ejection) click in aortic stenosis (AS) usually indicates a valvular AS, typically due
to a congenitally bicuspid aortic valve.
55) Some patients with AS may exhibit a dissociation of the systolic murmur into two components, with medium
frequencies transmitted to the base and high frequencies to the apex, almost mimicking MR (Gallavardin
phenomenon).
56) Findings arguing most strongly in favor of AS are a reduced/delayed carotid upstroke, a mid-to-late peak of the
murmur, a soft-to-absent A2, a palpable precordial thrill, and an apical-carotid (or brachioradial) delay. Conversely,
lack of radiation to the right carotid artery argues most strongly against AS. A normal rate of rise of the arterial
pulse argues also against the presence of significant AS, but only in the young.
57) The best bedside predictors for severity/clinical outcome of AS are (1) murmur intensity and timing (the louder
and later-peaking the murmur, the worse the disease); (2) a single S2; and (3) delayed upstroke/reduced amplitude of
the carotid pulse (pulsus parvus and tardus). Still, no single physical finding has both high sensitivity and specificity
for detecting severe valvular obstruction.
58) Presence of an audible S4 in AS reflects severe left ventricular hypertrophy (with a transvalvular pressure
gradient >70 mmHg), but only in younger patients (older subjects may already have a "normal" S4). Yet, a palpable
S4 always reflects severe disease.
59) In cardiac auscultation, the louder (and the longer) the murmur, the worse the underlying disease. The only
exception is severe aortic stenosis with decreased cardiac output.
60) Plateau mitral regurgitation (MR) murmurs are more likely to be rheumatic, whereas murmurs that start in
midsystole and "grow" into S2 are more likely to be due to either mitral valve prolapse or papillary muscle
dysfunction.
61) The acute MR murmur is often early systolic (exclusively so in 40% of cases) and is associated with S4 in 80%
of the patients.
62) Valvular aortic regurgitation (AR) tends to be loudest over the Erb's point (left parasternal area), whereas "root"
AR is loudest over the aortic area (right parasternal area).

DO NOT DISTRIBUTE - 20 -
Study Notes – Family Medicine & Physical ExamJames Lamberg 28Jul2010

63) The Austin-Flint murmur may occur in more than 50% of moderate to severe AR cases, usually requiring a
regurgitant volume of at least 50mL.
64) A palpable pulsus bisferiens usually reflects moderate to severe aortic regurgitation (with or without aortic
stenosis).
65) A difference in systolic pressure ≥60 mmHg between upper and lower extremities (Hill's sign) has high
specificity and a very high positive likelihood ratio for severe aortic regurgitation, but a sensitivity of only 40%. So
do a diastolic blood pressure ≤50 mmHg and a pulse pressure ≥80 mmHg.
66) Traube pistol shot sound(s) and Duroziez double murmur have sensitivity of 37-55% for AR and specificity of
63-98%. Neither predicts severity.
67) The alternate reddening and blanching of the fingernails, coinciding with each cardiac cycle and easily
visualized by lightly compressing the nail bed with a glass slide (Quincke's pulse), is one of the many peripheral
signs of AR, albeit a nonspecific and vastly discredited one.
68) You diagnose aortic regurgitation in diastole, but you assess its severity in systole (through the presence of a
flow murmur and possibly an ejection click). Conversely, you diagnose mitral regurgitation in systole, but you
assess its severity in diastole (through the presence of an S3 and possibly a diastolic flow rumble).
69) Tachypnea is so frequent in pulmonary embolism (92% of patients) that a normal respiratory rate argues
strongly against the diagnosis.
70) Unlike orthopnea, platypnea (an obligatory "supine respiration") is usually due to a right-to-left shunt. This can
be either intracardiac or intrapulmonary (typically bibasilar and common in cirrhotic patients-hepatopulmonary
syndrome).
71) Abdominal paradox has high sensitivity (95%) and good specificity (71%) for impending respiratory failure,
usually preceding arterial blood gases' deterioration.
72) Upward inspiratory motion of the clavicle in excess of 5 mm is a valuable sign of severe obstructive disease,
correlating with FEV1 of 0.6 L.
73) The distance between the top of the thyroid cartilage and the suprasternal notch (laryngeal height) is a strong
predictor of postoperative pulmonary risk if ≤4 cm.
74) The forced expiratory time (FET) is the best bedside predictor of the severity of airflow obstruction. FETo >6
seconds corresponds to an FEV1/FVC <40%. Conversely, FETo <5 seconds indicates an FEV1/FVC >60%.
75) Crackles (and rhonchi) that clear with coughing suggest airflow obstruction. Conversely, crackles that appear
after coughing (post-tussive crackles) argue in favor of tuberculosis.
76) Bronchial breath sounds reflect patent airways in a setting of absent alveolar air, with replacement by media that
better transmit higher frequencies, such as liquids or solids (consolidation). If unaccompanied by crackles, they
argue in favor of a pleural effusion.
77) Late inspiratory crackles can be detected by careful auscultation in 63% of young and healthy nursing students
(in 92% if using an electronic stethoscope with high-pass filtration).
78) Timing of crackles predicts the site of production, with early inspiratory crackles reflecting bronchitis, mid-
inspiratory crackles reflecting bronchiectasis, and late inspiratory crackles reflecting interstitial fibrosis or edema.
79) In asbestosis and idiopathic pulmonary fibrosis, the number of late inspiratory crackles correlates with disease
severity.
80) In patients with pneumonia, crackles and diminished breath sounds appear first; bronchial breath sounds and
egophony develop 1-3 days after onset of symptoms (i.e., cough and fever), and dullness to percussion (plus
increased tactile fremitus) occurs even later. This time lag usually allows for x-ray to preempt diagnosis, thus
making exam often irrelevant.
81) Wheezing on maximal forced exhalation has such a low sensitivity and specificity for asthma (57% and 37%,
respectively) to be completely unreliable for diagnosing subclinical airflow obstruction.
82) Wheezes are neither sensitive nor specific for airflow obstruction. Although unforced wheezing argues strongly
for chronic airflow obstruction, it can be absent in 30% of patients with FEV1 <1 L. It may also resolve in acute
asthmatics whose FEV1 remains at 63% of the predicted value. In fact, in status asthmaticus, wheezing is the least-
discriminating factor in predicting hospital admission or relapse.
83) Wheezing intensity does not correlate with severity of obstruction. Only pitch and length of wheezes are useful
predictors of airway narrowing. Higher-pitched and longer wheezes reflect worse obstruction.
84) Bowel sounds lack sensitivity and specificity for intestinal obstruction, being decreased or absent in only one
quarter of cases. Hence, they are clinically useless.
85) Lateral expansion of an abdominal mass ≥3 cm with pulsation suggests an abdominal aortic aneurysm. In cases
of small aneurysms (3-5 cm in diameter), the finding is very specific, with the few false positives usually reflecting a
tortuous aorta (yet, the finding is also poorly sen-sitive, detecting only one of five cases). In patients with large

DO NOT DISTRIBUTE - 21 -
Study Notes – Family Medicine & Physical ExamJames Lamberg 28Jul2010

aneurysms (>5 cm), sensitivity increases to four out of five patients. In fact, lack of expansile pulsation in a thin
patient should strongly argue against the presence of a large aneurysm.
86) Palpation of the liver edge is an unreliable way to estimate hepatic consistency. In fact, half of all palpable livers
are not enlarged, and half of truly enlarged livers are not palpable.
87) A pulsatile liver edge may represent transmission of aortic pulsations through an enlarged liver but usually
indicates one of two conditions: (1) constrictive pericarditis or (2) tricuspid regurgitation (TR). An inspiratory
increase in the magnitude of pulsations will be typical of TR (especially in held mid-inspiration/end-inspiration), but
not of constrictive pericarditis. Pulsatility in a setting of hepatomegaly instead is such a good indicator of
constrictive peri-carditis (present in 65% of patients) that its absence argues strongly against the diagnosis.
88) A painful arrest in inspiration triggered by palpation of the edge of an inflamed gallbladder (Murphy's sign) is a
good test for cholecystitis, with sensitivity and specificity of 50-80% (specificity usually a little higher than
sensitivity).
89) A palpable and nontender gallbladder in icteric patients strongly suggests that the jaundice is not due to
hepatocellular disease, but to an extrahepatic obstruction of the biliary tract, more likely neoplastic. Albeit not too
sensitive, this finding is highly specific.
90) In patients with splenomegaly, (1) concomitant hepatomegaly suggests primary liver disease with portal
hypertension; (2) concomitant lymphadenopathy excludes primary liver disease and makes instead hematologic or
lymphoproliferative disorders more likely; (3) massive splenomegaly (or left upper quadrant tenderness) also argues
in favor of a myeloproliferative etiology; and (4) Kehr's sign (referred pain or hyperesthesia to the left shoulder)
suggests impending splenic rupture.
91) Half of all patients with renovascular disease have a systolic murmur, whose significance depends on location
and characteristics. Overall, posterior murmurs are specific but not sensitive; anterior murmurs are sensitive but not
specific; anterior bruits (i.e., continuous murmurs) are both specific and sensitive.
92) Combining all bedside maneuvers provides a good bedside tool for the diagnosis of ascites, with overall
accuracy of 80%. Still, the amount of volume necessary for these maneuvers to become positive (500-1000 mL) is
much larger than that detected by ultrasound alone (100 mL).
93) Generalized adenopathy suggests a disseminated malignancy (especially hematologic), a collagen vascular
disorder, or an infectious process. Adenopathy presenting with fever usually suggests infection or lymphoma.
94) A palpable supraclavicular node carries a 90% risk of malignancy for patients older than 40 years, and a 25%
risk for younger patients.
95) A cranial nerve (CN) III palsy that spares pupils (i.e., ptosis and external rotation of the globe, but symmetric
and equally reactive pupils) suggests diabetes, but also vasculitides and multiple sclerosis.
96) In a meta-analysis of almost 2000 patients, the signs with highest likelihood ratios for predicting neurologic
recovery after a cardiac arrest were, at 24 hours: absent corneal reflexes (LR 12.9); absent pupillary reflexes (LR
10.2); absent motor response (LR 4.9); and absent withdrawal to pain (LR 4.7). At 72 hours, absent motor response
predicted death or poor neurologic outcome.
97) Many traditional findings in carpal tunnel syndrome, including Phalen, Tinel, and flick sign have low sensitivity
and limited or no value.
98) A positive straight-leg-raising test indicates nerve root impingement, usually by a herniated disk. It has high
sensitivity (91%) but low specificity (26%), thus limiting its diagnostic accuracy. The "crossed" straight-leg raising
test instead has low sensitivity (29%) but high specificity (88%). Hence, use them together.
99) A composite examination for anterior cruciate ligament (ACL) injuries has sensitivity >82% and specificity
>94%, with an LR of 25.0 for a positive examination and 0.04 for a negative one. Overall, a positive Lachman test
argues strongly in favor of ACL tear, whereas a negative is fairly good evidence against it. The anterior drawer is
the least accurate test.
100) A composite examination for posterior cruciate ligament (PCL) injuries has sensitivity of 91%, specificity of
98%, and LRs of 21.0 (for a positive exam) and 0.05 (for a negative one). The posterior drawer test is the most
reliable indicator, with mean sensitivity of 55%.
--------------------------------------------------------------------------------------------------------------------------------------------
Quick Osteopathic Review – OMT Review (3rd, Savarese)
--------------------------------------------------------------------------------------------------------------------------------------------
Somatic Dysfunction Definition
Somatic Dysfunction (SD) is impaired or altered function of related components of the somatic system: skeletal,
arthrodial and myofascial structures, and their related vascular, lymphatic, and neural elements.
Visceral (Viscerosomatic) Dysfunction is impaired or altered mobility or motility of the visceral system and related
fascial, neurological, vascular, skeletal and lymphatic elements.

DO NOT DISTRIBUTE - 22 -
Study Notes – Family Medicine & Physical ExamJames Lamberg 28Jul2010

Principles of Osteopathic Medicine


1) The body is a unit.
2) Structure and function are reciprocally inter-related.
3) The body possesses self-regulatory mechanisms.
4) The body has the inherent capacity to defend and repair itself.
5) When the normal adaptability is disrupted, or when environmental changes overcome the body’s capacity for self
maintenance, disease may ensue.
6) The movement of body fluids is essential to the maintenance of health.
7) The nerves play a crucial part in controlling the fluids of the body.
8) There are somatic components to disease that are not only manifestations of disease, but also are factors that
contribute to maintenance of the disease state.
Historical Dates
1828: Andrew Taylor Still born.
1874: A.T. Still 'flung the banner of osteopathy to the breeze.'
1892: First school opens in Kirksville.
1910: Flexner report causes many M.D. and D.O. schools to close.
1917: A.T. Still passes away at age 89.
1918: Fryette principles developed.
1961-62: The California experience, M.D. degrees purchased by ~2000 D.O.s for $65.
1963: D.O.s accepted by the civil service as medical officers.
1970: Most colleges change names to “College of Osteopathic Medicine.”
2010: 26 colleges of osteopathic medicine in 31 locations.
Diagnostic Criteria (TART) & Fryette’s Laws
T: Tissue Texture Changes Fryette Law I: N SBx Ry (opposite side)
A: Asymmetry Fryette Law II: F/E SBx Rx (same side)
R: Restriction Fryette Law III: Motion in one plane modifies all other planes
T: Tenderness
Various Tips & Facts
* Superior Facets (BUM BUL BM): Cervical is Backward, Upward, Medial. Thoracic is Backward, Upward,
Lateral. Lumbar is Back, Medial.
* Rule of 3s: T1-T3 spinous process at body. T4-T6 half below body. T7-T9 one below body. T10-12 one below,
half below, at body.
* Spondylolisthesis: Lateral X-Ray.
* Spondylolysis: Oblique X-Ray, pars defect at collar of Scotty Dog.
* Spondylolisthesis Grading: Grade 1 is 0-25%, Grade 2 is 25-50%, Grade 3 is 50-75%, Grade 4 is > 75%.
* Zink's Common Compensatory Pattern (CCP): L(O-A), R(C-T), L(T-L), R(L-S).
* Heel Lift Therapy: 1/16” for fragile/elderly, 1/8” for flexible, full amount for injured leg.
* Iliopsoas Flexion Contracture: L1-L2 F Rtoward SBtoward.
* Asthma Reflex: T2 on Left
* Vagus Nerve: Left C2, OA
* Cobb Angle (Scoliosis): Respiratory compromise > 50 degrees. Cardiac compromise > 75 degrees.
* Ferguson Lumbosacral: 25-35 degrees.
* Femur Q-Angle: 120-135 degrees.
Sympathetics
* Upper GI includes the Stomach, Liver, Gallbladder, Spleen, Pancreas, Duodenum.
* Upper GI (T5-T9) Greater Splanchnic Nerve and Celiac Ganglion.
* Middle GI includes Pancreas, Duodenum, Jejunum, Ilium, Ascending Colon, Proximal 2/3 Transverse Colon.
* Middle GI (T10-T11) Lesser Splanchnic Nerve and Superior Mesenteric Ganglion.
* Lower GI includes Distal 1/3 Transverse Colon, Descending Colon, Sigmoid Colon, Rectum.
* Lower GI (T12-L2) Least Splanchnic Nerve. Inferior Mesenteric Ganglion.
* Head & Neck: T1-T4. * Appendix: T12. * Gonads: T10-T11.
* Heart: T1-T5. * Kidneys: T10-T11. * Uterus & Cervix: T10-L2.
* Respiratory: T2-T7. * Adrenal Medulla: T10. * Penis & Clitoris: T11-L2.
* Esophagus: T2-T8. * Upper Ureters: T10-T11. * Prostate: T12-L2.
* Arms: T2-T8. * Lower Ureters: T12-L1.
* Legs: T11-L2. * Bladder: T11-L2.

DO NOT DISTRIBUTE - 23 -
Study Notes – Family Medicine & Physical ExamJames Lamberg 28Jul2010

Autonomics
Organ Parasympathetic Sympathetic
Eye Pupil Constricts (miosis) Dilates (mydriasis)
Eye Lens Contracts for near vision Slight relaxation for far vision
Glands (nasal, lacrimal, parotid,
Stimulates copious secretion Vasoconstriction for slight secretion
submandibular, gastric, pancreatic)
Sweat Glands Sweating on palms of hands Copious sweating (cholinergic)
Decreases contractility and Increases contractility and conduction
Heart
conduction velocity velocity
Bronchiolar Smooth Muscle Contracts Relaxes
Thin secretions, decreased Thick secretions, increased number of
Respiratory Epithelium
number of goblet cells goblet cells
GI Smooth Muscle Lumen Contracts Relaxes
GI Smooth Muscle Sphincters Relaxes Contracts
GI Secretion & Motility Increases Decreases
Arterioles, Skin & Visceral Vessels None None
Arterioles, Skeletal Muscle None Relaxes
Bladder Wall (Detrusor) Contracts Relaxes
Bladder Sphincter (Trigone) Relaxes Contracts
Penis Erection Ejaculation
Vasoconstriction of afferent arterioles,
Kidneys Unknown
decreased GFR, decreased urine volume
Ureters Maintains normal peristalsis Ureterospasm
Liver Slight glycogen synthesis Glycogenolysis (glucose into blood)
Uterus Body (Fundus) Relaxation Constricts
Uterus Cervix Constricts Relaxes
Parasympathetics
CN III (midbrain): Ciliary ganglion, pupils.
CN VII (pons): Sphenopalatine ganglion, lacrimal and nasal glands.
CN VII (pons): Submandibular ganglion, submandibular and sublingual glands.
CN IX (medulla): Otic ganglion, parotid gland.
CN X (medulla): Heart, bronchial tree, esophagus lower 2/3, stomach, intestine, liver, gallbladder, pancreas, kidney
and upper ureter, ovaries, testes, ascending and transverse colon.
S2-S4: Pelvic splanchnic, lower utter and bladder, uterus, prostate, genitalia, descending colon, sigmoid, rectum.
Cranial Nerves
Somatic Efferent
Cranial Nerve Exits Cranium Clinical Correlation
Dysfunction Afferent
CN I sphenoid, frontal,
Cribriform plate Altered sense of smell SVA
Olfactory ethmoid
CN II
Optic canal sphenoid, occiput Visual changes SSA
Optic
CN III Superior orbital Diplopia, ptosis,
sphenoid, temporal GSE, GVE-P
Oculomotor fissure accommodation problems
CN IV Superior orbital
sphenoid, temporal Diplopia GSE
Trochlear fissure
CN V1 Superior orbital Decreased sensation to
sphenoid, temporal GSA, SVE
Trigeminal fissure eyelid and scalp
CN V2 sphenoid, temporal,
Foramen rotundum Tic Douloureux GSA, SVE
Trigeminal maxillae, mandible

DO NOT DISTRIBUTE - 24 -
Study Notes – Family Medicine & Physical ExamJames Lamberg 28Jul2010

CN V3 Deceased sensation to
Foramen ovale sphenoid GSA, SVE
Trigeminal mandible
CN VI Superior orbital
sphenoid, temporal Diplopia, esotropia GSE
Abducens fissure
Internal acoustic SVE, GSA,
CN VII sphenoid, temporal,
meatus, Stylomastoid Bell palsy SVA, GVA,
Facial occiput
foramen GVE-P
CN VIII Internal acoustic sphenoid, temporal, Tinnitus, vertigo, hearing
SSA, SSE
Auditory meatus occiput loss
CN IX Decreased gag, decreased SVE, GSA,
Jugular foramen temporal, occiput
Glossopharyngeal taste GVA, GVE-P
Headaches, arrhythmias, SVE, GSA,
CN X
Jugular foramen temporal, occiput GI upset, respiratory SVA, GVA,
Vagus
problems GVE-P
CN XI Foramen magnum, Trapezius or SCM
temporal, occiput GSE
Accessory Jugular foramen tenderness
CN XII
Hypoglossal canal occiput Dysphagia GSE
Hypoglossal
Cranial Nerve Efferent & Afferent Contributions
GSE: General Somatic Efferent, motor to somatic voluntary skeletal muscles.
GSA: General Somatic Afferent, sensory to skin, joints, muscle and tendon receptor endings, nasal and oral cavity.
GVE: General Visceral Efferent, motor to smooth muscles (in general, skin, blood vessels), secretomotor to glands.
GVA: General Visceral Afferent, sensory to gut and derivatives, pharynx, blood vessels, glands and internal organs.
SSA: Special Visceral Afferent, special senses of vision, auditory sensation and vestibular apparatus.
SVA: Special Visceral Afferent, chemical senses of taste and by analogy smell.
SVE: Special Visceral Efferent, motor to voluntary skeletal muscles derived from branchial arches.
Neurologic Testing
Dermatome Deep Tendon
Vertebral Levels Muscle Strength Reflex Grading
Levels Reflexes
C6: Thumb, Radial T2/3 Body: Sternal 5: Full ROM, gravity 4/4: Brisk + Clonus
C5: Biceps
Nerve Notch + full resistance (UMN)

C7: 2nd/3rd Digits, T3 Spine: Scapula 4: Full ROM, gravity


3/4: Brisk C6: Brachioradialis
Median Nerve Spine + some resistance
C8: 4th/5th Digits, T4 Body: Sternal 3: Full ROM against
2/4: Normal C7: Triceps
Ulnar Nerve Angle gravity
T7 Spine: Scapula 2: Full ROM without
T4: Nipples 1/4: Decreased L4: Patella
Angle gravity
T6/7: Xiphoid T9 Body: Xiphoid 1: Slight contractility 0/4: Absent (LMN) S1: Achilles

T10: Umbilicus L4: Iliac Crest 0: No contractility


T12: Pubic Bone S2: PSIS
L1: Inguinal
Ligament
L4: Knee Caps
Overview of Osteopathic Techniques
* Direct Techniques: HVLA, Muscle Energy, Articulation
* Indirect Techniques: Strain/Counterstrain, Facilitated Positional Release, Balanced Ligamentous Tension
* Direct & Indirect: Myofascial Release
* HVLA: Also known as thrust technique. Used to remove articular restriction to motion—static asymmetry alone is

DO NOT DISTRIBUTE - 25 -
Study Notes – Family Medicine & Physical ExamJames Lamberg 28Jul2010

not a sufficient indication. Patient is positioned such that the dysfunctional joint engages the restricted barrier and a
quick thrust—high velocity—over a very short distance—low amplitude—is applied by the practitioner
* ME: A direct technique in which the physician engages the initial resistance to a barrier—the feather edge—and
asks the patient to provide a counterforce away from the barrier. Forces are generally light—excessive force is a
common error. The active contraction by the patient acts to mobilize a joint directly or by providing a postisometric
relaxation period whereby tissues can be further stretched.
* ART: Gentle, repetitive, and passive motion of a joint into its restricted. Examples include Spencer technique for
the shoulder and rib raising.
* MFR: A direct or indirect approach to releasing tension in muscles (myo)
and fascia (fascial). With a direct approach, tissue barriers are engaged rhythmically, or engaged and held. With an
indirect approach, tissues are taken to a place where the least amount of tension exists and held until an inherent
release occurs.
* Strain and CS: Treatment consists of positioning the patient to eliminate the tenderness and tension—a position of
ease—and holding this for 90 seconds. More than 200 specific points found anteriorly and posteriorly. Usually
discrete, fingertip size, tense, and edematous. Patient will typically wince because of exquisite tenderness of
significant points. Tenderpoints typically in tendons or muscle belly. They may also be found in other myofascial
tissues. If multiple tenderpoints are found, treat the most tender point first. Points are often away from a point of
trauma. For example, tenderpoints on the chest wall with back pain, or points in the antagonist muscle of a
traumatized muscle. If pain is associated with trauma, position of treatment matches the position of trauma and is
associated with maximal comfort.
* BLT: An indirect approach where tensions of the ligaments and membranous structures are balanced by joint
positioning. This allows an inherent release in the tissues. Active movement by the patient—often breath—is
commonly used to assist with the release.
* FPR: A form of indirect treatment—positional release—in which a facilitating force is added to decrease the
amount of treatment time needed. Facilitating force is compression and or torsion. Used for both tissue texture
change and joint restriction secondary to myofascial tension. Treatment always begins with putting joint into a
neutral position then adding a facilitating force.
Cranial Assessment
* Craniosacral flexion: midline bones flex, paired bones externally rotate.
* PRM = CNS + CSF + dural membranes + cranial bones + sacrum.
* Dural attachments: foramen magnum, C2, C3, S2.
* SBS: articulation of sphenoid with occiput.
* CRI = 10-14 cycles per minute.
* Factors decreasing CRI: stress, depression, chronic fatigue, chronic infections.
* Factors increasing CRI: exercise, systemic fever, OMT.
* Vault Hold: index on greater wing of sphenoid, middle on zygomatic processes of temporal, ring on mastoid
processes of temporal, pinky on squamous portion of occiput, thumbs cross on head.
* Torsion: AP axis, physiologic. Named for greater wing of sphenoid. If superior on right, right torsion.
* Sidebending/Rotation: 2 parallel vertical axes, physiologic.
* Sidebending of SBS cause sphenoid and occiput to rotate inferior to that side, hand feels fuller on that side (SBRL
or SBRR).
* Flexion/Extension: physiologic. Flexion is SBS cephalic, extension is SBS caudal.
* Vertical: 2 transverse axes, pathologic. Named for direction of sphenoid movement. Superior vertical strain, hands
move inferiorly.
* Lateral: 2 vertical axes, pathologic. Named for direction of sphenoid movement. Sphenoid right is right lateral
strain, index fingers point to left.
* SBS Compression: severely decreased CRI usually due to trauma, especially to the back of the head. a.k.a.
“bowling ball head”
* Craniosacral flexion, detailed: midline bones flex, paired bones externally rotate, sacrum moves posterior
(counternutation), ASIS widens, AP diameter of cranium decreases, transverse diameter increases.
* Fingers spread apart and abduct, sphenoid anteriorly rotates, basiocciput moves anteriorly and superiorly, foramen
magnum moves superiorly, condyles spread laterally.
* Occiput and sacrum move in the same direction. Sphenoid moves opposite of the sacrum.
Cervical Assessment & Counterstrain Points
* OA is Type I Mechanics: The side of the deeper sulcus is the OA rotation side. Flex and extend the occiput to
determine ease of motion.

DO NOT DISTRIBUTE - 26 -
Study Notes – Family Medicine & Physical ExamJames Lamberg 28Jul2010

* AA is Rotational: If Pt. rotates further right, AA RR.


* C3-C7 is Type II Mechanics: Posterior pillar is side of rotation and side bending.
* AC1 TP high on posterior edge of ascending ramus of mandible.
* AC2-C6 TP anterolateral tip of articular pillars (SCM).
* AC7 TP about 1 inch medial on superior clavicle.
* AC8 TP medial tip of clavicle.
* PC1 TP on inion and laterally on nuchal line.
* PC2-C6 TP interspinous ligaments between spinous processes.
Thoracic Inlet Assessment
* Thoracic Inlet is Type II Mechanics: Superior rib is side bending side.
* Inlet = C7, T1, T2, Rib 1.
* Example: Superior first rib on the left and anterior SC joint on the left, Inlet SR RR.
Thoracic Assessment & Counterstrain Points
* Example: T1-T4 posterior transverse processes on right, no change with flexion or extension, T1-4 N RR SBL.
* Example: T6 posterior transverse process on right, more symmetrical in flexion, T6 F RR SBR.
* AT1 TP at suprasternal notch.
* AT2-6 TP descend down sternum.
* AT7-12 TP are bilateral paired points.
* AT7 TP inferior to costal cartilage on either side of xiphoid.
* AT9-10 TP surround umbilicus.
* AT11 TP midway between AT10 and pubis.
* AT12 TP on iliac crests at midaxillary line.
* PT1-4 TP midline on lateral aspects of spinous processes.
* PT5-12 TP interspinous ligaments between spinous processes.
Rib Assessment & Counterstrain Points
* Inhalation Dysfunction: Ribs lag behind on exhalation (stuck up in inhalation). Ribs will be elevated anteriorly at
sternal border. Ribs will also be elevated along the length of the shaft, palpated at the midaxillary line.
* Inhalation Dysfunction: Key rib is inferior rib in group.
* Exhalation Dysfunction: Ribs lag behind on inhalation (stuck down in exhalation). Ribs will be elevated
posteriorly at rib angles.
* Exhalation Dysfunction: Key rib is superior rib in group.
* AR1 TP lateral to sternal angle of Louis.
* AR2 TP at midclavicular line 2nd rib interspace.
* AR3-6 TP at sternal border and midaxillary line rib interspaces.
* PR1-12 TP at posterior rib angles.
* Atypical Ribs: 1, 2, (10), 11, 12.
* Ribs 1-7: True Ribs. Ribs 8-12: False Ribs. Ribs 11-12: Floating Ribs.
* Ribs 1-5: Pump Handle Motion. Ribs 6-10: Bucket Handle Motion. Ribs 11-12: Caliper Motion.
* Rib 1 Attaches: Anterior & Middle Scalenes.
* Rib 2 Attaches: Posterior Scalene.
* Ribs 3-5 Attach: Pectoralis Minor.
* Ribs 6-9 Attach: Serratus Anterior.
* Ribs 10-11 Attach: Latissimus Dorsi.
* Rib 12 Attaches: Quadratus Lumborum.
Lumbar Assessment & Counterstrain Points
* Example: L1-L4 posterior transverse processes on right, no change with flexion or extension, L1-4 N RR SBL.
* Example: L3 posterior transverse process on right, more symmetrical in flexion, L3 F RR SBR.
* AL1 TP over or medial to ASIS.
* AL2 TP inferomedial surface of ASIS.
* AL3 TP lateral surface of ASIS.
* AL4 TP inferior surface of ASIS.
* AL5 TP body of pubic bone.
* PL1-UPL5 TP posterior aspects of transverse processes.
* LPL5 TP 2cm below superior medial surface of PSIS.
Sacrum Assessment & Counterstrain Points
* Seated Flexion Test (SeFT): +SeFT implies Sacroliliac > Iliosacral S.D.

DO NOT DISTRIBUTE - 27 -
Study Notes – Family Medicine & Physical ExamJames Lamberg 28Jul2010

* Forward Torsions: L on L, R on R
* L on L: R deep sulcus, L posterior ILA, +SeFT on R, Neg spring test.
* R on R: L deep sulcus, R posterior ILA, +SeFT on L, Neg spring test.
* Backward Torsions: L on R, R on L
* L on R: R deep sulcus, L posterior ILA, +SeFT on L, Pos spring test.
* R on L: L deep sulcus, R posterior ILA, +SeFT on R, Pos spring test.
* L Uni: L deep sulcus, L posterior ILA, +SeFT on L, Neg spring test.
* R Uni: R deep sulcus, R posterior ILA, +SeFT on R, Neg spring test.
* Bi Flexed: L&R deep sulci, L&R posterior ILA, Neg SeFT, Neg spring test.
* Bi Extended: L&R shallow sulci, Pos spring test.
* +SeFT side is opposite sacral axis.
* L5 Rotation is opposite sacral rotation.
* L5 Sidebending is same as sacral axis.
* Low ilium SI/anterior sacral TP on superior surface of pubes.
* Iliacus TP lower abdomen deep in fossa.
* Inguinal ligament TP at pubes attachment.
* Sacral Axes (CRAIN): Cranial (Superior Transverse, S2). Respiratory (Superior Transverse, S2). Anatomic
(Middle Transverse). Innominate (Inferior Transverse).
Pelvis Assessment & Counterstrain Points
* Standing Flexion Test (StFT): +StFT implies Iliosacral > Sacroiliac S.D.
* +StFT side determines side of dysfunction.
* Anterior Innominate Rotation: ASIS inferior, PSIS and ischial tuberosity superior. Ipsilateral leg appears longer.
* Posterior Innominate Rotation: ASIS superior, PSIS and ischial tuberosity inferior. Ipsilateral leg appears shorter.
* Superior Innominate Shear: Ischial tuberosity, ASIS, PSIS, and medial malleoli superior on same side.
* Inferior Innominate Shear: Ischial tuberosity, ASIS, PSIS, and medial malleoli inferior on same side.
* Abducted Pubes: pubic tubercles are wider than usual (lax symphysis).
* Adducted Pubes: pubic tubercles are closer than usual (compressed symphysis).
* Superior Pubic Shear: pubic tubercle is higher on side of +StFT.
* Inferior Pubic Shear: pubic tubercle is lower on side of +StFT.
* Inflare: shorter distance between ASIS and umbilicus on side of +StFT.
* Outflare: longer distance between ASIS and umbilicus on side of +StFT.
* Piriformis TP in middle of gluteus, check for externally rotated leg.
* Sacrospinous ligament divides the greater and lesser sciatic foramen.
Chapman Reflex Points
* Sinuses: Below proximal 1/3 clavicle.
* Ears: Above proximal 1/3 clavicle.
* Pharynx: Below 1st rib/manubrium junction.
* Pylorus: Sternum.
* Stomach Acid: 5th intercostal on left at costal cartilage.
* Stomach Peristalsis: 6th intercostal on left at costal cartilage.
* Liver: 5th intercostal on right at costal cartilage.
* Liver, Gallbladder: 6th intercostal on right at costal cartilage.
* Pancreas: 7th intercostal on right at costal cartilage.
* Heart: 2nd intercostal space at sternal border. T2 intertransverse space.
* Upper Lung: 3rd intercostal space at sternal border. T2 intertransverse space.
* Lower Lung: 4th intercostal space at sternal border. T3 intertransverse space.
* Adrenals: 1 inch lateral and 2 inches above the umbilicus. T11 intertransverse space.
* Kidneys: 1 inch lateral and 1 inch above the umbilicus. L1 intertransverse space.
* Appendix: Tip of 12th right rib.
* Colon: Iliotibial band in mirror image of colon layout. Cecum at right hip, transverse 1/3 colon at right knee,
transverse 2/3 colon at left knee, sigmoid at left hip.
* Bladder: Triangle around umbilicus.
* Prostate: Iliotibial band posterior margin.
Upper Extremity Assessment & Counterstrain Points
* Radial Head: Anterior radial head is a supination dysfunction, so restricted in pronation. Posterior radial head is a
pronation dysfunction, so restricted in supination.

DO NOT DISTRIBUTE - 28 -
Study Notes – Family Medicine & Physical ExamJames Lamberg 28Jul2010

* Fall forward causes posterior radial head.


* Fall backward causes anterior radial head.
* Supraspinatus TP superior to spine of scapula deep in the supraspinatus muscle.
* Long head of biceps TP is over the tendon.
* Short head of biceps TP is inferolateral to the coracoid process.
* Subscapularis TP on anterior surface of scapula.
* Latissimus dorsi TP deep to axial on medial surface of humerus.
* Lateral elbow TP proximal to radial head.
* Medial elbow TP proximal to coronoid process.
* Rotator Cuff (SItS): Supraspinatus, abduction. Infraspinatus, external rotation. Teres minor, external rotation.
Subscapularis, internal rotation.
* Shoulder Internal Rotation: Bra Strap. Shoulder External Rotation: Shampoo Hair.
* Shoulder Issue (RAIN): Rotator Cuff Tear. Adhesive Capsulitis, AC Joint. Impingement, Instability. Neck, Nerve.
* Brachial Plexus: Roots, Trunks, Divisions, Cords, Branches.
* Branches (MARMU): Musculocutaneous, Axillary, Radial, Median, Ulnar.
* Winged Scapula: Long thoracic nerve palsy.
* Erb-Duchenne: Upper brachial plexus palsy (C5-C6).
* Klumpke: Lower brachial plexus palsy (C8-T1).
Lower Extremity Assessment & Counterstrain Points
* Fibular Head: Anterior fibular head will have increased anterior slide with decreased posterior slide. Posterior
fibular head will have increased posterior slide with decreased anterior slide (e.g. inversion ankle injury).
* Anterior talus (posterior tibia) prevents dorsiflexion, is plantarflexion dysfunction. Shallow sulcus under the tibia.
* Posterior talus (anterior tibia) prevents plantarflexion, is dorsiflexion dysfunction. Deep sulcus under the tibia.
* Anterior tibia on talus, joint prefers dorsiflexion. Shallow tibial sulcus.
* Posterior tibia on talus, joint prefers plantar flexion. Deep tibial sulcus.
* Trochanteric TP on lateral, posterolateral, and posteromedial surfaces of greater trochanter.
* Lateral patella, medial patella, lateral meniscus, and medial meniscus TP at expected locations.
* Gastrocnemius TP lateral and medial attachments of the muscle in the lower popliteal fossa.
* Medial hamstring TP on muscle near distal attachment.
* Lateral hamstring TP on muscle near fibular head.
* Calcaneal TP at distal end of calcaneus on sole of foot.
* Lateral ankle TP below the lateral malleolus.
* Talar TP on anteromedial ankle deep to talus.
* Femoral Triangle: Inguinal Ligament, Adductor Longus, Sartorius.
* Toward NAVeL: Nerve (femoral). Artery (femoral). Vein (femoral). empty. Lymph nodes (inguinal).
Orthopedic Testing: Upper Extremity
* Adson Test: Compression of the subclavian artery.
* Allen Test: Collateral circulation of the hand.
* Apley Scratch Test: Evaluate the range of motion of the shoulder.
* Apprehension (Crank) Test: Detect chronic shoulder dislocation.
* Drop Arm Test: detect tears in the rotator cuff muscles.
* Empty Can Test: Detect tears of the supraspinatus tendon or muscle.
* Finkelstein Test: DeQuervain tenosynovitis, abductor pollicis longus and extensor pollicis brevis tendons.
* Load & Shift Test: Shoulder instability, anterior or posterior.
* Phalen Test: Carpal tunnel syndrome.
* Posterior Apprehension Test: Posterior shoulder instability or dislocation.
* Speed Test: Bicipital tendinitis.
* Sulcus Sign: Inferior shoulder instability.
* Tinel Sign: Carpal tunnel syndrome.
* Yergason Test: Bicipital tendinitis.
Orthopedic Testing: Lower Extremity
* Anterior Drawer Test (Knee): Anterior cruciate ligament.
* Anterior Drawer Test (Ankle): Anterior talofibular and calcaneofibular ligaments.
* Apley Compression Test: Knee meniscal injury.
* Apley Distraction Test: Knee ligamentous injury.
* Babinski Test: Upper motor neuron dysfunction.

DO NOT DISTRIBUTE - 29 -
Study Notes – Family Medicine & Physical ExamJames Lamberg 28Jul2010

* Barlow Test: Hip stability.


* Erichsen Test: Sacroiliac pathology.
* Galeazzi (Allis) Test: Congenital hip dislocation, age 3-18mo.
* Homan Sign: Deep vein thrombophlebitis.
* Lachman Test: Anterior cruciate ligament.
* Ludloff Sign: Traumatic separation of the lesser trochanter of the femur.
* McMurray Test: Meniscal tears.
* Ober Test: Iliotibial band/fascial lata dysfunction.
* Ortolani Test: Congenital hip dislocation, newborn.
* Patrick (FABER) Test: Hip joint pathology.
* Posterior Drawer Test: Posterior cruciate ligament.
* Thomas Test: Contraction of the iliopsoas muscle.
* Thompson Test: Ruptured Achilles tendon.
* Trendelenburg Test: Gluteus medius muscle, superior gluteal nerve.
* Valgus Stress Test: Medial (tibial) collateral ligament.
* Varus Stress Test: Lateral (fibular) collateral ligament.
--------------------------------------------------------------------------------------------------------------------------------------------
Kaplan Videos (2001) – Preventative Medicine with Dr. Jacob Levy, MD
--------------------------------------------------------------------------------------------------------------------------------------------
Screening Tests
* 39yo female comes to your clinic very concerned about her risk of developing cancer. When questioned further
she attests to a family history of colon cancer. Her father was diagnosed at the age of 43 and her mother was
diagnosed with breast cancer at the age of 52. The patient states that she is sexually active with multiple partners and
has not seen a physician since a motor vehicle accident that occurred 15 years ago. She denies any symptoms at this
time and physical exam is normal. At the end of the interview, she asks what you recommend.
* What is screening? What does it mean to screen for specific diseases? How do we screen, why do we do it, when
should we do it, and when should we consider it?
* Screening is testing an asymptomatic population for the presence or absence of disease.
* Advantages to screening would be improving mortality by catching cases early enough to intervene.
* Risks of screening include iatrogenic complications (perforation during colonoscopy). Meaning the complication
only occurred because we screened for colon cancer.
* There is a risk of labeling a patient as ill, for example if you tell a patient they are hypertensive they may call in
sick to work more often. Patient may be more prone to leaving work and seeking disability.
* We screen when we can intervene effectively. We don’t screen for ALS because there is no intervention we can do
to affect the disease, so there is no point in determining who will get the disease. So in order for a screening test to
be useful there has to be an intervention shown to improve mortality in a patient.
* We only screen when the test has been shown to decreased all-cause mortality. We do not screen based on an
improvement in survival.
* Another requirement for the screening test is that the patient has to agree to the intervention we would do if the
test comes back positive. Example would be a 45yo male who refuses to surgery. Then there is no point in doing a
colonoscopy because the patient will refuse the intervention if the colonoscopy is positive.
Cancer Screening
* Three types of cancer that have a clear recommendation for screening are: colon cancer, breast cancer, and
cervical cancer.
* There are no clear recommendations for screening of lung cancer even for patients who are very high risk. This is
because no intervention or screening test would reduce all-cause mortality.
* There are no clear recommendations for screening for prostate cancer; various groups recommend different
options but the American College of Physicians does not have a clear statement for the screening of prostate cancer.
Their recommendation is to discuss screening on a case-by-case basis.
* There is evidence for colon cancer screening. Recommendations are annual fecal occult blood test from the age
40yo. Age of 50yo, get colonoscopy every 10 years or flexible sigmoidoscopy every 5 years.
* If the patient has a family history of colon cancer, we start 10 years earlier (colonoscopy at age 40 or 10 years
younger than the lowest age of onset amount the relatives). Family history is first degree relative who gets colon
cancer < 60yo. If patient’s father had colon cancer diagnosed at age 43, then she should be screened at age 33.
* With family history, patient should get colonoscopy every 3-5 years.
* Breast cancer screening is done by mammogram, physician breast exam, and patient self-exam. There is no clear

DO NOT DISTRIBUTE - 30 -
Study Notes – Family Medicine & Physical ExamJames Lamberg 28Jul2010

evidence that self examination improves mortality. Why do we recommend it then? Because it is cheap and easy.
Recommendation is monthly self breast exam starting at age 20 done 7-8 days after end of menses.
* Physician breast exam is recommended annually starting at age 40. Mammogram recommended annually from 50-
75 years of age. Most physicians between age 40-50 recommend mammogram every 1-2 years. Some newer
evidence is pushing toward mammograms starting at age 50, not 40. Can get a physician breast exam every three
years between the age of 20 and 40.
* Cervical cancer screening is done at age 21. Previous guidelines were age 18 or sexual activity. Pap smear is the
major modality of screening and should be done annually until there are three negative Pap smears. After three
negative Pap smears in a row, you can screen every 2-3 years until age 65.
Traveler Health
* 44yo executive comes to your clinic prior to traveling to Thailand for business. He has no significant past medical
history and is only here to see you because his company will not let him travel until he is seen by a physician. The
patient appears agitated and demands your recommendations immediately.
* Consider vaccinations: hepatitis A, hepatitis B, malaria prophylaxis, traveler’s diarrhea.
* For hepatitis B vaccination, ask patient if they plan to engage in any sexual activity with the local population. If
they will, they should be immunized against hepatitis B. If patient will be functioning in close proximity to the
population, such as working as a nurse, they should be immunized.
* For hepatitis A vaccination, immunize if patient is going to an endemic area. Endemic areas for hepatitis A are any
non-industrialized nation (rural India, Middle East, North Africa, South Africa). If patient is planning to eat and
drink in those areas, they should be vaccinated.
* Generally you want to see patients for travel medicine one-month prior to them taking the trip.
* Say patient shows up and needs hepatitis A but is two weeks from their trip. You can give immunoglobulin in
addition to the vaccination. So you give them passive immunity with Ig, give vaccination, then give second shot of
hepatitis A vaccination when they return from the trip.
* Hepatitis A immunoglobulin given if patient is departing within two weeks.
* For a patient traveling to a malaria endemic area, what is the prophylaxis? For Caribbean or Central America, you
give chloroquine. Mnemonic is “C” for Caribbean, Central America, Chloroquine. For everywhere else, they should
get mefloquine due to chloroquine resistance. Alternative regimen is atovaquone and proguanil for patients going to
a mefloquine-sensitive or chloroquine-resistant area.
* For traveler’s diarrhea, you can give loperamide to help with diarrhea if there is no fever and no blood in the stool.
If the patient has fever with blood in stool, give fluoroquinolone (ciprofloxacin). You can give the patient the
medication for the trip, but make sure they know only to take it if they have a fever and blood in their stool.
* Caution patients not to eat unwashed vegetables and fruits, even in a fancy restaurant. Should only drink bottled or
boiled water to avoid traveler’s diarrhea.
* Enterotoxigenic Escherichia coli (ETEC) is the most common cause of traveler’s diarrhea.
Immunizations
* 52yo man comes to your clinic for a health maintenance evaluation. His recent colonoscopy showed no evidence
of carcinoma. His recent serum fasting glucose, serum cholesterol, and blood pressure measurements were all within
normal limits. The patient has a history of smoking and continues to smoke two packs per day. He was diagnosed
with COPD three years ago. What is your recommendation for this patient?
* For this patient, we should do fecal occult blood testing if he has not had one within a year. There is no indication
for colonoscopy because he just had one. You should address this patient’s immunization status.
* In the U.S., up to 70,000 adults die a year secondary to preventable infectious disease. The simple act of providing
an immunization when recommended can seriously improve the mortality and morbidity of a lot of patients. You
should consider this in any patient you see, regardless of what specialty you choose for your career.
* In patients who are pregnant or immunosuppressed, we generally do not give live vaccines.
* Live vaccines are varicella, MMR (measles mumps rubella), yellow fever, polio (live not given anymore).
* Varicella is given in any adult who has no history of varicella (chicken pox) infection.
* MMR is given in any adult born after 1956, give them a booster vaccination.
* MMR is the only live vaccination that is safe in HIV-positive patients, given they have no symptoms.
* Pregnant females should never get a varicella or MMR vaccine.
* Hepatitis A is universally recommended, but high risk is chronic liver disease and would not be able to tolerate an
acute infection or if they are traveling to an endemic area.
* Hepatitis B is for patients exposed to blood, young sexually active patients, or those with chronic liver disease.
* Tetanus given every 10 years in the adult population after their initial series of shots.
* Influenza in universally recommended annually. Previous guidelines were after age 50 or young if chronic disease.

DO NOT DISTRIBUTE - 31 -
Study Notes – Family Medicine & Physical ExamJames Lamberg 28Jul2010

* You give influenza every year because the virus is different each year, so previous immunity may not cover.
* Pneumococcal vaccine given at age of 65 if no co-morbidities. Given before 65 if co-morbidities, especially
asplenic, immunosuppressed, smokers, asthma, COPD, emphysema, American Indians, Alaska Natives.
* If patient receives their pneumococcal vaccine before the age of 60, they should be re-vaccinated at the age of 65.
* Any smoker age 19-64 should receive the pneumococcal vaccine with smoking cessation counseling.
* Any patient with chronic lung problems age 19-64 should get the pneumococcal vaccine.
Smoking Cessation
* 25yo man comes to clinic for evaluation of stuffy nose and fever. You diagnose pharyngitis and give him
antibiotics. Over the course of the interview, the patient states that he smokes three packs of cigarettes per day and
has been doing so for the past seven years.
* Smoking is the cause of about 1 in every 5 deaths in the United States.
* If you can get your patient to stop smoking, you have accomplished a tremendous amount in the prevention of
lung disease, cancer, and other smoking-related causes of death.
* There are 5 steps to smoking cessation (5 A’s): Ask, Advise, Attempt, Assist, Arrange.
* Ask the patient at every visit about their smoking.
* Advise the patient to stop smoking at every single visit. Just the advice from a doctor has been shown to increase
the rate of smoking cessation. You have a positive influence on your patients.
* Attempt to identify those patients who want to quit.
* Assist the patient by setting a quit date and providing pharmacologic support, either patch or gum or bupropion.
* Arrange for follow-up with the knowledge that most patient require several quit attempts to finally stop. At that
point, discuss why they relapsed and what can be done differently. Encourage them to try again, try again, and try
again. The more attempts a patient makes, the more likely they are to quit.
* The importance of smoking cessation cannot be overstated.
Diabetes, Cholesterol, & Hypertension Screening
* 45yo man comes to your office anxious about his health. When questioned further he states that the source of his
anxiety is his family medical history. Five years ago his mother was diagnosed with diabetes and high cholesterol.
He is worried about his health and risk of heart disease. Physical exam is within normal limits.
* Screen for diabetes when you have risk factors for diabetes. Risk factors are obesity and family history.
* Diagnosis is random glucose > 200 or fasting blood glucose > 126 on two occasions.
* There is no clear recommendation for screening for diabetes unless risk factors are present.
* Cholesterol screening for males starts at age 35 and females from the age of 45.
* There is no clear recommendation for how often a screening cholesterol should be taken. If a patient has multiple
risk factors for atherosclerotic disease, they should be followed more closely with cholesterol screening.
* Hypertension screening should occur at every visit and at least every two years.
Alcohol Abuse Screening
* 55yo man comes to your office for evaluation of a sore throat. Upon further discussion you know that the patient
was recently fired from his job and is having marital problems at home. The patient has no significant past medical
history and physical exam is within normal limits. He attests to drinking 3 shots of whiskey every day after work.
* CAGE questionnaire is used to screen for alcohol abuse. Cut down, Annoyed, Guilty, Eye opener.
* Have you ever tried to cut down on your drinking?
* Do you get annoyed when people talk about your drinking?
* Do you ever feel guilty about the amount of alcohol you drink?
* Have you ever taken a drink as an eye-opener to wake you up in the morning?
* A CAGE score of 2/4 is considered a positive screen.
* How much is too much? Men may be at risk for alcohol-related problems if their alcohol consumption exceeds 14
standard drinks per week or 4 drinks per day, and women may be at risk if they have more than 7 standard drinks per
week or 3 drinks per day.
* A standard drink is one 12-ounce bottle of beer, one 5-ounce glass of wine, or 1.5 ounces of distilled spirits.
* In the above 55yo, the patient is having trouble functioning in their normal social context. They are having
problems at work and at home. They are using alcohol as an escape, showing signs of drug abuse.
Physical Abuse Screening
* 27yo woman is brought to the Emergency Department complaining of right arm pain. When asked how she
sustained the injury she states that she fell down the steps in front of her house. The patient appears anxious and
nervous. On physical exam you notice various lacerations that are 2cm wide on her buttocks.
* As physicians, we can prevent injury and death by counseling our patients about safe practices and behavior.
* Any patient under the age of 65, the number one cause of death is traumatic injury. It can be not wearing a seatbelt

DO NOT DISTRIBUTE - 32 -
Study Notes – Family Medicine & Physical ExamJames Lamberg 28Jul2010

in the car, not wearing a helmet on a bike, playing with firearms, drinking and driving, physical abuse.
* An effective way to screen for abuse is to ask patients simply and clearly: Have you ever been threatened by
somebody close to you? Have you ever been hit or bitten by anyone close to you? Up to 10% of the time, patients
will answer yes and will identify themselves. Simply by asking the question, you can do a lot of good.
* The area of preventive medicine is not very glamorous. It is not about giving thrombolytic therapy to save a
patient who is having an MI. It is about giving medication to prevent the MI in the first place.
* Preventative medicine is less glamorous, but much more effective.
* Address health issues, cancer screening, immunization status, counsel about alcohol abuse, ask about physical
abuse, counsel about safe behavior, counsel about smoking cessation.
--------------------------------------------------------------------------------------------------------------------------------------------
Kaplan Videos (2001) – Medico Legal Cases with Dr. Steven Daugherty, MD
--------------------------------------------------------------------------------------------------------------------------------------------
* It is important to have a set of rules to apply to these types of questions. You will likely see questions you have not
studied before, so having rules to apply to any question will make them easy to answer.
* The answer on an exam may not be what you’ve seen in real life. The exam is not about what everybody does in
real life, they are about what everybody should be doing in real life.
* Note: This is not legal advice on how to practice medicine.
Karen Ann Quinlan Case (Substituted Judgment)
* Karen Ann Quinlan case gave rise to the rule of substituted judgment.
* Karen Ann was a young woman living in NJ, in her early 20s and was out driving with her friends looking for fun
one Saturday evening. Another driver was out drinking, ran a stop sign, and crashed into Karen Ann’s car. Several
of her friends died and Karen Ann was left in a persistent vegetative state.
* Doctors examining Karen Ann said they could keep her on life support almost indefinitely but she is unlikely to
regain consciousness and certainly not much quality of life.
* When the father heard this prognosis, he said lets disconnect life support and let nature take its course.
* This case ended up in court because there was no clear rule; it should never end up in court today.
* The court listened to the doctors and what the father said and went with their wishes. Life support was terminated.
Miraculously, Karen Ann lived 3-4 months without life support then subsequently died.
* The important part is that the father got to make the decision because if Karen Ann had sat up and had a “lucid
moment” she would have said what the father said. The father did not get to make the decision because he was the
dad, he got to make the decision because he was the best representative of what Karen Ann’s own wishes were.
* The father was the best window to the wishes of the patient.
* It is not about next of kin. It is never about next of kin. Next of kin does not matter. Substituted judgment matters.
* A man is brought into the Emergency Department with his partner, a homosexual companion with whom he has
been living for the past 10 years. The man is at the end-stages of HIV dementia. He has all the symptoms including
lack of cognitive capacity and inability to express his wishes in any way. If you treat this man aggressively, you can
keep this man alive for about 2 months. Without aggressive treatment, the man will die in about two weeks. How
aggressively should you treat this patient?
* The partner says they do not know, but remembers the patient saying that he did not want to drag this thing out. So
the partner thinks we should not do anything and let nature take its course.
* Then the patient’s family come into the hospital. Mom, father, aunt, uncle, family dog Spot. The whole family in
unison says do everything possible to keep their loved one alive “as long as possible.”
* It is not clear still. Now you find that because of the man’s lifestyle he has been estranged from his family for
several years. Now it is clear to listen to the partner and not the family. (In real life, it may be risky to go against
family wishes to keep a patient alive even if they have the worst substituted judgment for the patient.)
* Answer choices that say go to court are generally not the right answer. The basic rule on the exam is you as the
physician makes the call.
Brother Fox Case (Best Interest Standard)
* Eichner v Dillon (Brother Fox Case) gave rise to the rule of best interest standard.
* This case is about someone who was never competent. So in the eyes of the law, there was no substituted
judgment because the patient never wanted anything. Guessing at wishes when functionally the patient never had
wishes is pretty difficult. Substituted judgment does not apply here.
* Best interest standard means you act as a dispassionate rational decision maker weighing the benefits and burdens
of any problem, doing what any rational person would do. If benefits are great and burdens few, you act. If benefits
few and burdens great, you do not act.
* So you’re saying I do not know what you in particular want, but I’m doing what most people would want in this

DO NOT DISTRIBUTE - 33 -
Study Notes – Family Medicine & Physical ExamJames Lamberg 28Jul2010

same situation. This is the best interest of the patient and no one else.
* If the decision maker has strong philosophical beliefs that preclude a decision being made, they are considered
irrelevant when using the best interest standard. Personal beliefs cannot come into play.
* The person making the decision is irrelevant, because any person using the best interest standard should come to
the same decision.
* In general, physicians do not make decisions for patients. This is an exception. It is not because of your position,
your expertise, it is because you are just as good as anybody else with the best interest standard.
* If you’re stuck with the best interest standard, think about laying all the evidence out in front of a random
community jury of 12 people. What they would tell you to do, is what you do.
Infant Doe Case (Parents Cannot Withhold Life-Saving Treatment)
* Infant Doe Case gave rise to the rule of parents cannot withhold life- or limb-saving treatment from their child.
* Infant Doe was the third child born to a happy mother and father. This child was born with a whole list of medical
problems including gross mental retardation, Down syndrome, tracheoesophageal fistula.
* Unless the fistula was operated on, the child would certainly die.
* The surgery team said they could do the surgery, but even if it was successful the child would live out its life in a
custodial state without any of the things we think of as human life. The surgery team said they were 50/50 at best on
delivering the best outcome for this child.
* The parents heard this and said not to operate, letting nature take its course. The hospital doctors agreed.
* The local county prosecutor disagreed. He said it was his job to defend the helpless, which this child is. He sued to
force treatment and lost. He appealed to the Indiana Supreme Court and lost again. He was getting together his
appeal to go to the U.S. Supreme Court but the child died.
* So are there clear extraordinary circumstances where a parent can withhold life-saving treatment, clearly there are.
If this case troubles you, angers you, disturbs you, you are not alone.
* The court used the best interest standard, saying the benefits were few and burdens great so no surgery.
* So, parents cannot withhold life- or limb-saving treatment from their children. The exception is when the best
interest standard says otherwise.
Roe v Wade (Patients Make Medical Decisions)
* Roe v Wade gave rise to the rule of patients (not doctors) make medical decisions.
* Roe v Wade is known to the public as the abortion legalizing decision, but this is wrong on many levels. Abortion
has always been legal in some of the United States. Roe v Wade said states could not make abortion illegal in the
first trimester. States may, if they wish, decide if abortion is legal or illegal in the last two trimesters.
* Abortion is not an exam issue because the laws that govern abortion differ radically across the 50 states.
* This case was about the rights of mother’s own body versus the rights of an unborn child. This goes to the heart of
who makes medical decisions in the United States, and that is patients not doctors. This is fundamental and differs
from many other countries in the world.
* If you think you are better educated and better informed thus can make decisions for patients you will be getting a
lot of questions wrong because it is not the doctor’s decision to make.
* Your job is to come to the patient like a waiter. Here’s the menu, I’d recommend the triple bypass today. In a
democracy the patients make the laws. Most people would like to decide for themselves when faced with a decision.
If you’re a doctor and don’t like what the patient decides, too bad, it’s their decision to make.
* If the mother is ill and you want to do something for the sake of the child, courts have held that you cannot unless
the mother says it is alright. If the mother is bleeding profusely and the fetus is in the 9th month and will die without
a blood transfusion, but the mother says no blood, do you give it to her? No, she said no.
* Another case involved a fetus that had the cord wrapped around its neck three times, mother refused C-section.
Court said its mother’s right even though team thought the baby and the mother would die. Team prepared and there
was a happy ending where both lived. So again, you believe the baby will die without intervention it does not matter
because it is not your decision to make.
Tarasoff Case (Duty To Warn And Protect)
* Tarasoff v Regents of U of CA gave rise to the rule of duty to warn and duty to protect.
* Student at the University of California went to the university counseling center because they were feeling distress.
During the counseling cession, student said I’m so mad at another student I’m going to go out and kill them. Then
the student left the counseling cession.
* The counselor was so concerned that she called the police and said she had a student that was threatening to harm
another student. The police picked up the student who visited counseling, but the student said I was just talking
during counseling no big deal. Since no crime was committed the police let the student go.
* The student then went out and killed the person they said they were going to kill.

DO NOT DISTRIBUTE - 34 -
Study Notes – Family Medicine & Physical ExamJames Lamberg 28Jul2010

* The counselor, the counseling center, and the University were found liable for millions of dollars. The counselor
did one thing she should do (call the police) but did not warn the potential victim.
* There is a duty to warn by letting people know they are in danger and a duty to protect by acting to stop the harm
from happening if you can at all do so.
* You are seeing a middle-aged man for essential hypertension for several years. He comes in for a standard
checkup, you check his BP, do a physical, and adjust his medications. When he’s getting dressed and about to leave,
he pauses, looks back, and says, “Doctor, I don’t know why I’m telling you this but I have to tell someone, when I
leave here I’m going to go kill my wife” then walks out the door. You notice a bulge that looks like a large revolver
in his pants pocket. What should the physician do next?
* If you said go for the phone, you just got the question wrong. You should be following the patient out the door.
But doesn’t the patient have a gun? Rule for the exam, we don’t care about you (the doctor). The exam doesn’t care
if you get sued, shot, maimed, they only care about the patient. You’re not expected to tackle the patient and wrestle
the gun away, but you’re expected to be able to talk the patient down from this situation.
* Say patient gets away. Now call the police. Then call the wife. But what do you say to the wife when you call?
* Tarasoff 2 Case says if you tell the wife that her husband is coming home to kill her, and she greets him at the
door with a shotgun and kills him, then you the doctor are liable.
* So clearly what you say to the wife on the phone is important. What goal do you want to achieve here. I want her
to be safe and I want to know that she is safe. You can do that by getting the patient to your office.
* So say, “I need to talk to you about something very urgent and I need you to come right now. Drop everything you
are doing and come to my office immediately.”
* Why not tell the patient to go to the police station? Because you don’t know that she actually went to the police
station. Why not tell the patient to go to the neighbor and wait for the police? Because you don’t know what
happened yet. If the potential victim is at your office you can go with her to the police together.
--------------------------------------------------------------------------------------------------------------------------------------------
Kaplan Videos (2001) – Medico Legal Rules with Dr. Steven Daugherty, MD
--------------------------------------------------------------------------------------------------------------------------------------------
Rule #1: Competent Patients Have The Right To Refuse Medical Treatment
* If the patient is incompetent, you still need permission but not from the patient, from the surrogate or guardian.
You don’t make decisions, patients do.
* Here are some “exceptions” people use: Life is the greatest good, once lost it is irreplaceable. I must intervene
because if I don’t the patient will die and that’s the greater good. I must intervene because to not intervene is like
helping the patient commit suicide. Protection of third parties, I must save this woman’s life else her children will all
be orphans and for their sake I must intervene. I took an oath, based on this oath I must intervene.
* These were listed because none of them work on the exam or in real life. If you thought these were good
exceptions to the rule, then you’re wrong. A competent patient has the absolute right to refuse treatment.
* The patient will die without a blood transfusion in an hour. The alert patient says, “I don’t want a blood
transfusion.” The outcome of this case is that the patient dies. They have the right to refuse.
Rule #2: Assume Competence
* It is not the job of the patient to prove to you that they are competent. You must have clear court designation or
behavior evidence that they are incompetent. If you don’t, then they are competent.
* If you are presented with a case and wondering if you’re in the gray area, “is this patient competent or not?” then
they are competent. They are either completely incompetent, or are considered competent. There is no gray area.
* Schizophrenic patient for 15yrs, in and out of treatment facilities for the past 5 years, he is on a large does of
haloperidol several times a day. Is he competent? Of course. What does diagnosis have to do with competency, that
is a medical decision. Competence is a legal decision and you are not qualified to determine competence.
* Woman has Alzheimer dementia for the past 5 years. She has been in a nursing home care facility for the past 15
months. Is she competent? Yes.
* What blood alcohol level do you need to determine incompetence? None, competence is not a blood test. You look
at the behavioral evidence.
* Patient comes in and is drunk. Are they competent? You assume competence.
* Suppose you’re working late in the ED and an ambulance roles in, the patients arm is crushed with nerves sticking
out and blood shooting out. You realize this person repeatedly slashed their arm with a knife in a suicide attempt.
The man is conscious, as you’re trying to stop the bleeding he says “No, let me go, I want to die.” Do you let him
die? No. Trying to kill yourself is clear behavioral manifestation of incompetence.
* The schizophrenic patient is talking to you, eyes roll back, starts to giggle, then falls to the floor and rhythmically
beats his head against the floor. Competent? No. Any random 10 people you picked off the street would say this

DO NOT DISTRIBUTE - 35 -
Study Notes – Family Medicine & Physical ExamJames Lamberg 28Jul2010

person is incompetent.
* If you think someone is incompetent, you can’t just do whatever you want. You need permission from the closest
surrogate or guardian.
Rule #3: Avoid Going To Court
* For exams, going to court is a bad answer. The examiner figures if they watch you make a decision they are
learning something about you. If you toss problems off to someone else, they learn nothing about your capacity. So,
make the call.
* You have a child that is 7yo with juvenile onset diabetes. The child is not an emergency, but if not treated they
will die in several months. The patients, because of religious beliefs, refuse to have the child treated. Instead they are
going to take the child back home and pray for the child’s recovery with their religious community. This is the
exception when you go to court. The court will take over medical guardianship of the child and make medical
decisions. It is not because we don’t like the parent’s religion, that is irrelevant. It is because we don’t believe in the
parent’s decision. Parents are not allowed to withhold life- or limb-saving treatment from the child.
* Say this is an emergency but the parents say no. Do you act. Yes of course, they cannot withhold.
* Do you need a court’s approval to terminate life support. No, make the decision. The court says, “Look, you
passed all your exams, right? You know what the rules are and we’re really busy. Stop bothering us and make the
decision. Leave the hard ones for the court.”
Rule #4: Use Subjective Standard, Then Substituted Judgment, Then Best Interest Standard
* Use subjective standard (advanced directive), then substituted judgment, then best interest standard.
* Advanced directive means if the patient is unable to communicate right now, then we’re going to do what the
patient said before they were in this state. This is clear historical verbal or written wishes.
* Say the patient never said what they wanted. Now we use substituted judgment (Karen Ann Quinlan). You never
said what you wanted but I know you, so I’ll tell people what you want.
* Say no one knows the patient or what they want, then we use the best interest standard (Brother Fox). This is the
last step. We sit and weight what most people would want and make a decision based on that.
* It’s been a busy night in the ED and at the end of the shift an 87yo woman is brought in by her adult daughter who
has been caring for her at her own home. The 87yo woman is in an apparent coma, persistent vegetative state.
Examination and neurological consultation suggests that this woman is unlikely to ever regain consciousness but
could probably be kept alive on life-support for several years. You wonder how aggressively to treat this patient so
you start talking to the daughter. The daughter is visibly distressed and breaks down crying, you wait, give her a
tissue, she composes herself, and you start again. The daughter says I don’t want to make this decision, I don’t
know, I don’t know. You ask what she thinks her mother would want. She says mother never talked about what she
wanted, if I had to decide I’d say do nothing aggressive because mom didn’t want to be a burden on people, but I’m
just not sure. Now comes the mother’s adult son, who has been estranged, living 2000 miles away, but raced to the
hospital upon hearing about the condition. He got there via his private jet and helicopter, landing on the hospital. He
is able to pay for this lifestyle because he does well as a malpractice attorney suing physicians for bad medical
decisions. The son points out that you haven’t done anything to treat his mother, you haven’t even done the standard
protocol interventions that are done in these circumstances, and he insists that you begin treating. Now what?
* So what options? Listen to the daughter, listen to the son, sit with them both and come to a decision, go to court,
talk to the hospital ethics committee, talk to the hospital legal committee, call the chief attending physician. The
answer is listen to the daughter. We do not have clear subjective standard because the patient did not write anything
down. Now we’re at substituted judgment and who do you believe has the best substituted judgment, daughter.
* Say the son pulls out a letter written by the mother. It says she wishes they will do everything to help her. Now we
have clear subjective standard, so who cares what the son or daughter say, this is the patient’s wishes.
Rule #5: The Physician May Rely On Advanced Directives
* If the patient is incompetent, the physician may rely on advanced directive.
* Advanced directive can be oral.
* Prior to going into surgery, you are going through the procedure with the patient and making sure they understand
the details because you believe this enhances patient recovery. As you’re going over the procedure one last time, the
patient grabs your arm and looks into your eyes seriously, saying, “Doctor, I’ve thought about this and I know
you’re going to do your best but if something happens during surgery I don’t want to be kept alive on life-support,
just pull the plug.” You assure the patient everything will be fine and you’ve done this hundreds of times before.
Then something horrible happens during surgery and they end up in a coma. The family hears about this condition
and identifies the country’s leading specialist in this kind of comatose patient. They come to you saying that the
specialist is flying in first-class from across the country. At this point, what should the physician do. Answer is pull
the plug. Wait wait, there were no witnesses for this conversation. You also forgot to chart the conversation because

DO NOT DISTRIBUTE - 36 -
Study Notes – Family Medicine & Physical ExamJames Lamberg 28Jul2010

you were so busy so there is no documentation. Yes, still pull the plug. Wait, won’t you get sued? Absolutely. Not
only will you get sued but you’ll lose big time, your house is gone, your car is gone, your license is gone, you’re
flipping burgers. If you’re thinking how do I protect myself legally on the exam, you’re not going to get the answer.
* When should you have a conversation with the patient about a living will? The answer is as soon as you begin to
be their physician. You don’t have conversations about these things when they are at the threshold of difficult
medical procedures. You don’t have this conversation when they hear the word cancer, they’re not listening. You
have the conversation early when they are able to think and reflect, making the best decision for themselves.
* Health Power of Attorney (POA) is a person speaking with the patient’s voice, just like the patient.
* So patient goes under for surgery and says pull the plug if something happens. They go into a coma and the POA
says do everything. You do everything because it is the last conversation with the patient that matters and the most
recent conversation (by the POA, same as patient) was keep them alive.
* Say living will reads “Do everything to keep me alive” and POA comes in and says pull the plug. What do you
do? You pull the plug.
* The Health POA has on the spot facts about what should be done. The dates on the documents are irrelevant. If the
Health POA was designated 2 years ago and the willing will was 2 weeks ago, we listen to the Health POA. Health
POA is like a trump card.
Rule #6: Feeding Tubes Can Be Withdrawn At The Patient’s Request
* Feeding tube is considered a medical treatment and can be withdrawn at the patient’s request.
* A competent patient has the right to refuse even life-saving food and hydration.
* The willpower needed to starve yourself to death is incredible and very few people have that.
* Cruzan case, feeding tube was removed, patient continued to live.
* What about anorexia nervosa? A 15you refuses food and water. What do you do? Feed them. Why? The patient is
a minor so what they say does not matter. Most cases of anorexia are in the teen years.
* Say patient is 19yo with anorexia nervosa and is refusing food and water. Outcome is go to court. Anyone would
recognize this is a clear syndrome, the court will say she is not competent, and they will take over medical
guardianship of the patient.
Rule #7: Do Nothing To Actively Assist The Patient To Die Sooner
* What is acceptable and not acceptable for euthanasia? Not doing something at the patients request is acceptable.
Not giving food, medications, blood, that is fine because it is the patient’s decision. Actively doing anything to
speed up the time of death is unacceptable, don’t do this. No you can’t leave morphine syringes on the table and
walk out of the room.
* One common question, “Doc, if I knew I had a way out it would help so much more with the pain. If you could
just write me a prescription for something…” Do not do this. Don’t even give the patient pain medication where you
know you’re getting close to the respiratory depression threshold and causing death.
* Give the patient as much pain medication as they can stand. Do more than you think you should; if you say the
patient is terminal and there is nothing more than can be done you’re lying. You can always give them comfort and
pain relief. But, do this in a way that does not kill the patient.
* In real life, if you do an intervention like give pain medication and they end up dying, this is considered dual
intervention or dual consequence and we forgive you. On the exam, don’t go there.
Rule #8: The Physician Decides When The Patient Is Dead
* You’ve been treating a patient for several months doing all the standard and even some non-standard
interventions. Yet, the patient continues to be in a persistent come and does not show any response. You believe
there is no hope of improvement and your colleagues agree. You believe any further intervention with this patient is
a waste of valuable medical resources. You wish to terminate treatment with the patient. The family says no, they
want you to keep treating even though they agree there is no benefit from the intervention. What do you do, keep
treating. Decisions to terminate treatment (or start treatment) are not yours to make.
* You’ve been treating a coma patient for several months without any improvement or chance of recovery. A review
of the data shows the EEG over the past 48hrs has been flat line. You confront the family and they say they know a
specialist who is not flying in to consult on their loved one. What do you do? Call time of death. You do not get to
decide when to terminate treatment, but you get to decide when you have a patient and a corpse. You had flat-line
over 48 hours when you only need it over 24 hours. The patient is dead. Make the call, terminate life support.
* It is not futile in the sense that the treatment is not working, it is futile in the sense that the patient is dead and that
is your decision to make.
Rule #9: Never Abandon A Patient
* If a patient wants you as their doctor, do you have to take them? No. But once you take a patient you cannot
abandon them. What is the patient is completely annoying. “Unless you improve your manner with me I can no

DO NOT DISTRIBUTE - 37 -
Study Notes – Family Medicine & Physical ExamJames Lamberg 28Jul2010

longer be your doctor.” That’s wrong, just deal with it.


* A patient makes a sexual advance? Deal with it, don’t refer. What if the patient just can’t pay? Answer is keep
treating and don’t get paid. It is about making sure the patient gets what they need, not money.
* You have a diabetic patient who is poorly controlled. You tell them about nutrition, how to lose weight, how to
take their meds. After 9 months they have done none of it. You say, “Look, if you can’t do anything I say how can I
be your doctor?” Wrong, you are abandoning the patient.
Rule #10: Always Obtain Informed Consent
* I’m going to give my patient a chewable baby aspirin that they could get over the counter. Do I need informed
consent? Yes. Any intervention requires that the patient be told information and you get consent.
* For informed consent, you need 5 parts and if any one is missing then you don’t have consent.
* 1) Tell the patient the nature of the procedure, what are you going to do.
* 2) Tell the patient the purpose of the procedure.
* 3) Tell the patient the benefits of the procedure.
* 4) Tell the patient the risks of the procedure.
* 5) Tell the patient what the alternatives are including doing nothing.
* HMOs use to have “gag” clauses that forced physicians to only describe interventions that would be covered by
the HMO or that the HMO wanted; they are illegal because they violate informed consent.
* Attending physician sends you in to get informed consent on a patient. They say do the usual but don’t talk about
this one thing because the patient never understands and it usually scares the patient. The research shows it hasn’t
been effective anyway. Do you tell the patient about that option? Yes you do else you do not have informed consent.
* You have to lay out all the options and make sure the patient understands it. What if the patient doesn’t speak
English but you have a family member right there to translate? Do not use this family member because they won’t
translate the way you need. Have the hospital translator come in.
* Say you ask the patient if they understand the situation, explaining all the details. You fall silent and the translator
talks for a while. The patient then talks for a while. The translator looks at you and says, “The patient says Yes.”
Now what? Look at the translator and say it is very important that they say everything the patient has said, will you
do that now for me please. Deal with the issue presented right there.
* You come into the room with all the paperwork the patient has to sign for informed consent. The patient is
watching TV and says, sure doc I’ll just sign, and clearly is not listening to you. The correct next step is to tell the
patient that you will come back later. The patient does not understand what they are signing.
* Exception: Emergency. You don’t need informed consent, act now and talk about it later.
* Exception: Waiver by the patient. This is where the patient agrees with full forethought not to get informed
consent. Example would be research. I’d like to tell you the benefits and risks of this medication, but we don’t
know, that’s why we’re doing the research. Or, we get waivers prior to surgery. We think you have one thing wrong
but if it’s not then we’d like to take care of whatever it is.
* You get consent for appendicitis and go in for surgery. While in surgery you notice the appendix looks fine, but
there is a vicious-looking tumor there. Do you take it out? No, you don’t have consent. Do you go talk to the
husband in the waiting room to get consent? No, the husband does not get to make the patient’s decisions. You
should close, wake up the patient, and get full informed consent before doing the second surgery.
* Say you have complete informed consent, then the patient says wait I changed my mind. You no longer have
informed consent.
* Say you wheel the patient into surgery and they get the usual course of benzodiazepines. You start general
anesthesia and the surgeon begins to scrub. A young student nurse says, “I don’t know if it matters but the patient
said wait wait right before we put them under the general anesthesia.” What should you do? You should err on the
side of ensuring consent, so wake the patient up, wheel them out, get full informed consent. In real life the surgeon
would likely say “I didn’t hear anything” then cut.
Rule #11: Special Rules Apply With Children
* Anybody under the age of 18 is a minor, meaning they are not an adult and are not competent.
* Exception: Emancipated Minor. Usually age 13 and up living by themselves, acting as an adult, supporting
themselves as an adult, and contributing to society as an adult. In this case, we treat them as adults so they get to
make business decisions and medical decisions for themselves.
* 14yo shows up in a car accident, you think the arm is broken. You ask for parental consent and he says, “I haven’t
seen those bastards in years.” Translation is this is an emancipated minor. What if they lied? Don’t go paranoid on
me, that’s not your problem. You are acting as a doctor not a detective.
* Marriage emancipates a minor. Armed Services emancipates a minor.
* Things that do not make you emancipated in general are pregnancy and having a child. These differ by state, so a

DO NOT DISTRIBUTE - 38 -
Study Notes – Family Medicine & Physical ExamJames Lamberg 28Jul2010

pregnant minor may be emancipated during pregnancy but no longer emancipated after giving birth depending on
the state.
* What about a 16yo who dropped out of high school but has a job and is contributing to the family and paying all
their bills. Are they emancipated? Yes.
* Partial emancipation applies to any child (usually age 16-17) that gives them the right to make medical decisions
in four area only. They are substance abuse, prenatal care, treatment for STDs, and birth control.
* These exceptions exists because they are obvious public health concerns. We’re worried that a 16yo would be
scared to tell his parents about her STD and simply not get treated. If he can get treatment for himself without his
parents knowing then he is more likely to get treatment and thus we protect the public.
* 16yo female patient that you’ve had for 10 years. She comes for a physical exam prior to summer camp. You find
nothing remarkable on exam and as you’re filling out the forms she says, “When I go to summer camp I’m going to
go with my new boyfriend and we’ve decided to start having sex. I want to do the responsible thing and don’t want
to become pregnant so can you give me birth control pills and not tell my parents.” What should the physician do or
say next? The right answer is not to write her a script and send her on her way. Yes, you can write her a script and
no you shouldn’t tell the parents. The correct answer is counseling, advice, use of barrier protection for STDs.
Rule #12: Parents Cannot Withhold Life- Or Limb-Saving Treatment From A Child
* A mother and her 17yo girl are brought in by ambulance to the hospital after an automobile accident. Both are
alert, conscious, and bleeding profusely. Unless both receive a transfusion within an hour they will both die within
two hours. The mother refuses blood transfusion citing her religious beliefs. The daughter refuses blood transfusion
as well, citing her own participation in these religious beliefs. You take the daughter away from the mother to make
sure she isn’t being influenced and the daughter continues to refuse. Do you transfuse the mother? No. Do you
transfuse the daughter? Yes.
* A mother brings in a child with a deep bleeding cut on her forearm. The cut would require about 20 stitches,
although you could take care of it with butterfly closure bandages and compresses. Yet that would leave an unsightly
scar that might possibly impede the use of her arm in her elderly years. The mother refuses you permission to stitch
the arm. Do you stitch? No. This is not life- or limb-threatening.
Rule #13: Anything That Varies Widely Across The United States Cannot Be Tested
* Abortion issues differ widely so do not get tested.
* 12yo girl comes in and wants an abortion but doesn’t want you to tell her parents. Can you do the abortion? No,
there is no state that you can do an abortion on a 12yo without parental consent.
* They get around this by backing the age down to the point where everyone is consistent.
Rule #14: Good Samaritan Laws
* Good Samaritan laws are an attempt to make the world a better place, they try to get physicians to do what they
ought to do by stopping to help people in non-medical settings when those people need care.
* The law does not say you have to stop to help. Say you see a horrible accident on the highway, blood everywhere
and people screaming. You drive by. Will you get in trouble? No.
* The law is meant to take away a barrier that people use to not stop and help. In the United States, for better or
worse, you can be sued any day by anybody for anything. If that’s the case, why would you stop to help?
* What the Good Samaritan law does is shields you from that liability if you stop to help. If you stop to help and do
what is reasonable, you are shielded from legal liability even if every patient you touch dies.
* You are shielded as long as four things hold true:
* 1) Your actions are within your level of competence. If you’ve been a dermatologist for decades then opted to do
on-the-spot brain surgery at the scene of an accident, that’s probably over the line.
* 2) You use accepted procedures. This is not the time to try new things.
* 3) You have to stay if you stop. You can’t be giving CPR then say, “Ow my back, I’m outta here.” You are
committed until you are relieved by competent medical personnel (e.g. ambulance).
* On the exam, you should get in the ambulance and ride with them to the hospital because it is ethically the right
thing to do. Is it possible that something could happen in the ambulance where you could save the patient and the
EMTs/Paramedics could not? Yes, so don’t abandon your patients.
* 4) You cannot get paid or compensated. Once you get paid it is fee-for-service and all the regular liability laws
will come into play.
* You splint a guy’s broken arm and he pulls out a wad of cash to give you as thanks. Don’t take it. If you do, it’s
fee-for-service and not Good Samaritan. How about he says he owns the new restaurant downtown and you should
visit with your buddies for a free meal. Don’t take the meal. How about a package shows up at your door two weeks
later and it’s a Rolex watch with a card from the patient. Don’t accept the watch. The amount does not change the
answer. Do not give this up casually, keep the shield.

DO NOT DISTRIBUTE - 39 -
Study Notes – Family Medicine & Physical ExamJames Lamberg 28Jul2010

* It’s 7:15pm and your office closes at 7pm. Your office sign is turned to closed and you are doing paperwork. You
haven’t locked your doors yet and a man stumbles into your office with a large butcher knife stuck in his abdomen.
He passes out in the lobby. Do you have to help this man? Yes, he made it into a medical setting.
* It’s 7:15pm and you just left your office. You see a man outside on the sidewalk with a large butcher knife stuck in
his abdomen. Do you have to help him? No, he’s not in a medical setting. This isn’t what we want you to do of
course, but it is how the legal system works.
* You are flying in an airplane and the stewardess comes over the speaker to say, “Is there a doctor, we need a
doctor in first class.” Do you have to go help? No.
* You are not forced to help but the Good Samaritan law tries to remove the barrier if you want to help.
Rule #15: Confidentiality Is Absolute
* You don’t tell anybody anything about any patient period.
* Never discuss a patient anywhere where you can be overheard. Err on the side of not telling.
* A husband and wife always come in together for the husband’s visits. One day the husband comes alone because
the wife is doing errands. She calls later to ask about the visit because she is usually there. Can you tell her about the
visit? No. You cannot even tell her if the husband kept the appointment. That fact that he was there or not is
confidential and the fact that you are someone’s doctor or not is confidential.
* You get a subpoena to come testify about a patient next Thursday. Do you show up next Thursday. Yes. Do you
testify? No, a subpoena is easy to get and you should not burst patient confidentiality. Say because of patient
confidentiality you cannot discuss the matter. You can only tell if there is a threat of harm to the patient or others.
Not only can you break confidentiality in that exception, you must break confidentiality.
* Your male patient has been diagnosed with Gonorrhea and is being treated. He says please don’t tell my wife
because he got this during an affair and he doesn’t think his marriage would survive the strain. What do you do?
This is a mandatory reportable public disease, so you have to report it to public health. The notion that the wife will
not find out is not in the cards. The wife will find out from public health. The discussion should be about how we
tell the wife because she will find out. The husband says he’ll go tell the wife and leaves. Is that the right answer?
Probably not. How about you call and say, “This is Dr. so-and-so, you don’t know me but have you ever considered
being tested for Gonorrhea?” Nope, wrong answer. Best way to do this is bring them both together into the office,
she finds out, she gets informed and gets advices from you.
* Husband has HIV, does the wife find out? Yes, spousal notification is the law.
* With any infectious disease, encourage the patient to bring in their significant other so you can tell them about the
disease. Voluntary notification is generally recommended, where the patient notifies their partner. In some states, the
physician has the right to call the partner, in some other states they prefer the physician not tell.
* A man makes a living as a bus driver but you find an eye condition as part of a physical exam. You think this eye
condition would make driving a bus dangerous. He says please don’t tell my boss. Do you tell the boss? Yes,
prevent harm from happening.
Rule #16: Patients Should Be Given A Chance To State DNR And Physicians Should Follow Them
* If the DNR says no pulmonary resuscitation, can you do cardio resuscitation? Yes. The DNR says what you cannot
do and you can do whatever else. Generally it means don’t call for the crash cart.
* If someone has DNR, does that mean you don’t perform scheduled surgery? Of course you do the surgery.
* If someone has DNR, do you stop the antibiotic drip? No, you continue regular medical care. You just do not do
heroic measures to bring them back from the brink.
Rule #17: Doctors Don’t Commit The Mentally Ill, Courts Commit
* Doctors do not make the decision to commit a mentally ill patient and that patient is entitled to a jury trial before
they are committed.
* Committed means the patient is assigned to a locked treatment facility and cannot come and go as they please.
* This is not a jail, treatment must be available.
* The patient can refuse treatment. I go to talk to my schizophrenic patient today and give them their haloperidol,
but they say, “No, I don’t want that.” They said no, you can’t force it. You go ahead and treat if they let you treat.
* A committed patient still has the right to transact business, get married, get divorced, vote, drive on a day pass.
Rule #18: Physicians Can Detain
* Emergency detention is allowed in most states for 48 hours in some states and at most 72 hours in most states.
* Can you hold a patient against their will if it will prevent harm from happening to themselves or others? Yes
absolutely, this is therapeutic privilege.
* Children are not allowed to be detained unless they are an imminent danger to self or others, unless they are
unable to care for their daily needs, or unless the parents have no control over the child.
* Example would be a child that starts fires and breaks windows everywhere they go and the parents cannot do

DO NOT DISTRIBUTE - 40 -
Study Notes – Family Medicine & Physical ExamJames Lamberg 28Jul2010

anything about it, then we can protect the child and others by detaining them.
Rule #19: If A Health Care Professional Puts Anyone At Risk, Remove Them From The Risk
* Examples would be infectious disease. The doctor has tuberculosis, do you want them seeing patients? No.
* This covers substance abuse too. Your coworker has an alcohol abuse problem, should they be treating patients?
No, take whatever action necessary to prevent that from happening.
* A colleague is very depressed. Should they be treating patients? No, take action to stop them from treating.
* You think your colleague is completely incompetent. Should they be seeing patients? No. How do you make that
happen? If you’re a resident, you can go to the program director and discuss it. If it’s a freestanding physician out in
practice then you go directly to the state licensing board to make the report. Your loyalty on this exam is to the
patient. It is not to yourself or other physicians.
* If a physician is HIV positive, do they have to tell their patients? It depends on if it is relevant. Say you’re a
radiologist or psychiatrist, then it doesn’t matter if you have no patient contact. How about a surgeon? Yes. That
would be a risk to discuss during informed consent.
* A fellow physician has a substance abuse problem. You report them to the licensing board. Then what? Refer to
the local treatment facility? No. The right answer is you personally take the physician on as a patient. You go with
them to the facility.
Rule #20: Focus On The Best Ethical Conduct
* Think “What would Mother Teresa do in this instance?” Whatever you decide is the answer you choose.
* It’s not about reflecting on real life, it’s not about what the lawyers want you to do, it’s about what the exam-
writers want you to do.
--------------------------------------------------------------------------------------------------------------------------------------------
Kaplan Videos (2001) – Epidemiology with Dr. Steven Daugherty, MD
--------------------------------------------------------------------------------------------------------------------------------------------
Screening Tests
* Screening tests are about secondary prevention and are about prevalence (not incidence).
* Sensitivity involves how good a test is at finding disease in a population; detection of disease.
* 100% sensitivity means every person with disease was found by a test.
* Rule: True on top for every calculation.
* Sensitivity (Sn) = True Positive / (True Positive + False Negative)
* A sensitivity of 80% is pretty good, chance would be 50% so at 80% we’re not doing too bad.
* Specificity involves the detection of health.
* Specificity (Sp) = True Negative / (True Negative + False Positive)
* Mnemonic: for sensitivity (SN) the false Negative is on the bottom, for specificity (SP) the false Positive is on the
bottom. SPIN and SNOUT: Specificity rules disease IN, Sensitivity rules disease OUT.
* It is very difficult to get a test with high sensitivity and specificity. In practice, we pick tests that either have high
specificity or have high sensitivity, but usually not both.
* Screening should start with a test that has good Sensitivity, then confirm with a test that has good Specificity.
* So which do you want, good specificity or good sensitivity? Well it depends on what you’re trying to achieve.
* Suppose there is a horrible infectious disease out there but we have a great way to treat it. Would you want a test
that is sensitive or specific? Here we want sensitivity to make sure we catch all the cases.
* Suppose there is a disease we can treat, but the treatment is a little dangerous and pretty painful. In this case we
want specificity so that we aren’t subjecting healthy people to the painful treatment.
* A patient comes to you and says they think they got HIV. You do the screening test and it comes back positive.
Now you have to tell the patient the news. As a side note, you should both be sitting and you should be talking to the
patient either face-on or slightly offset, but not side-by-side. So what do you tell the patient? You say, the test says
you have HIV. The patient will ask, are you sure? You say: the positive predictive value for the test.
* Every time you get a positive lab result you should ask yourself if you believe the test result.
* Positive Predictive Value (PPV) = True Positive / (True Positive + False Positive)
* You could get a lab result back on an exam and be asked what should the physician do? If you calculate the PPV
and decide the test is garbage, it will completely change what you do with the test result.
* A female patient comes in to see you for fertility trouble. Now she returns because she missed a period and wants
a pregnancy test. The pregnancy test comes back negative. You have to deliver the news and the patient asks if you
are sure? You say: the negative predictive value for the test.
* Negative Predictive Value (NPV) = True Negative / (True Negative + False Negative)
* Accuracy = (True Positive + True Negative) / N, N is the total for the table
* If prevalence is 50%, which is common, then accuracy is (Sensitivity + Specificity) / 2

DO NOT DISTRIBUTE - 41 -
Study Notes – Family Medicine & Physical ExamJames Lamberg 28Jul2010

* Suppose that disease prevalence increases. What effect does that have on sensitivity? It stays the same. Sensitivity
and specificity are properties of the test, so they will not change with changes in prevalence.
* Suppose that disease prevalence increases, what happens to positive predictive value? PPV increases. Say you
have a group of people where 50% have a disease and you have another group of people where disease is one in a
million. If you got a positive from each group, which would you believe? The 50% group. How about a negative?
You would believe the one in a million group. So prevalence has to affect the PPV and NPV.
* This is the reason we tend not to screen entire populations because by definition your prevalence is going to be
low, thus your PPV will be bad. So we focus our screening on areas where we think there is a higher likelihood of
disease because our PPV will be higher.
* The bimodal “double hump” diagram is used to help decide where a screening test cutoff should be made.
* The bimodal diagram is meant to show a hump for healthy people and a hump for diseased people. The majority of
the hump to the left of the middle line is the true negatives (healthy) and to the right is the true positives (diseased).
The mistakes are in the little triangles that overlap into the larger hump sections. The ones to the left on the negative
side of the line are false negatives and the ones to the right on the positive side of the line are false positives.
* A new screening test is applied to a representative 1000 people in a population. Based on the data presented in the
table, calculate the requested screening test measures. The first thing to do when encountering a 2x2 table is to make
sure everything is in the right order. Focus on the denominator first. If asked about sensitivity, just look for the
number divided by TP + FN. For PPV, look at the number divided by TP + FN. For accuracy, look at the total (N)
for the denominator, then look at TP + TN for the numerator. Prevalence is disease out of total population.
* The CDC would like to decrease the cutoff for a screening test from X to Y. X was at the point where the two
bimodal curves crossed. What will happen? True positive tests will increase and false positive tests will increase.
PPV will decrease since we have more false positives. The accuracy of the test will be reduced. The sensitivity is
concerned with classifying diseased people, so the sensitivity will increase. If the number of positive test results
increases and those people go to follow-up, there will be an increase in the number of follow-up patients.
* Now say there is a little bit of cross-over between two bimodal curves and they ask what the sensitivity of the test
is. If you split a curve in half, you would be at 50% sensitive. If you drew a horizontal line just outside the tail of the
curve, you would be at 100% sensitivity. So, at the point where the curves cross you are between 50% and 100%
sensitive. If the overlap tail is small, you’re probably closer to 100% sensitive.
* There is a screening test with sensitivity of 80% and specificity of 90%. Disease prevalence is 10%. What is the
PPV for this test? You have to construct a 2x2 table, assume you have 1000 people (pick a nice round number). Now
work backwards, so you have 100 diseased people (10% prevalence) and 900 healthy. Out of 100 diseased people,
TP must be 80 and FP must be 20. Then TN is 90 and FN is 810. PPV is then 80 / 170, so just below 50%.
Research Designs: Observational Studies
* The developers of the exams know not everyone is going to become a researcher. They realizes however that all
physicians will be consumers of research and should be able to interpret research studies. Does this study have bias?
Is what this pharmaceutical representative telling me accurate? What about a patient that shows up with a study they
printed off the Internet after seeing it in the news? You should be able to understand if the study applies to the
patient and why.
* Whenever presented with a question on research, ask yourself if this is an observational study (writing down what
happened) or an experimental study (an intervention such as medication or surgery).
* Observational studies are the first type we do in medicine. They are done in a specific order.
* Case report is an observational study that is a description of a single patient. N = 1 in a case report. This is how we
realize something is possible in medicine. The first time we learned about HIV was a single case report.
* Case series is an observational study that is a bunch of patients with something in common. This helps us get a
clear list of symptoms or signs for a disease. However, there is no control group.
* Cross-sectional study is simply a prevalence study. This observational study is to figure out how many people in
an entire population have the disease right now. We do not get incidence or a measure of causality. We have
chickens and eggs but no temporal sequence so we don’t know what came first.
* Case-control study is cases of disease compared with control (no disease). We get these people together and look
back into time retrospectively for the presence or absence of some precursors. The goal is to determine what the
precursors are for the disease. We do not get a measure for the prevalence of disease since we just pick a group of
people. We also do not get incidence because we are starting with disease. We do get a measure of causality.
* There is a sudden and widespread cholera epidemic. No common water source can be found. How do you do a
study to figure out the problem? You do a case-control study, get people together and interview them about
absolutely everything you can think of. Then you look between the two groups for differences and those differences
are likely precursors to disease. When this study was done, all the patients with cholera were found to love mangos.

DO NOT DISTRIBUTE - 42 -
Study Notes – Family Medicine & Physical ExamJames Lamberg 28Jul2010

The mangos are washed at the point of shipping, so that was the source. People were told not to eat mangos and the
cholera epidemic was stopped.
* Cohort study is an observational study that is the most elaborate, time-consuming, and expensive. Thus, we do it
last after the case report, case series, cross-sectional study, and case-control study. A cohort study is the exact
opposite of a case-control study. It starts with people who have risk factors then follows them forward in time to see
who does or does not develop disease. This gives you a measure of incidence of disease. You do not get a measure
of prevalence. You do get the best measure of causality because you are in the right temporal sequence with cause
then effect afterwards.
* Cohort studies give incidence data. We analyze data as relative risk (division) and attributable risk (subtraction).
* Relative Risk = Incidence In Exposed / Incidence In Unexposed
* Relative risk is a ratio of incidence rates. Say infant mortality for African Americans is 20 per 1000 live births.
Say infant mortality for Caucasians is 10 per 1000 live births. These are incidence rates. What is the relative risk for
African Americans versus Caucasians? RR is 20/10 = 2, twice as likely. What about Caucasians versus African
Americans? RR is 10/20 = 0.5, half as likely.
* Relative risk answers the question how much more likely.
* Attributable risk answers the question how many more cases.
* Attributable Risk = Incidence In Exposed - Incidence In Unexposed.
* In the previous case, 20 - 10 = 10. So 10 more African American babies died per 1000.
* You should be much more concerned with attributable risk. You don’t treat probabilities, you treat cases.
* Relative risk is important from a statistical point of view, it gives you statistical significance, p-value.
* Attributable risk is how you get a gauge of clinical significance, how many more cases you are likely to see.
* Number Needed To Treat (NNT) is how many people you have to treat (medication, surgery, etc) to make one
person better. This is linked to the incidence rate; it is the inverse of the incidence rate.
* If the incidence is 1 in 1000, we have to treat 1000/1 = 1000 people to make 1 better.
* If incidence rate is 10 per 1000, NNT is 1000/10 = 100.
* NNT = 1 / Incidence
* We analyze case-control study data with odds ratio.
* Looking at a 2x2 table does not tell you the type of study, you have to see where they got the data from. Say we go
out right now and look at who has and who does not have lung cancer, then who is or is not a smoker. What type of
study is this? Cross-sectional. Say we look at smokers and non-smokers then follow them for a few years to see who
gets lung cancer. That is a cohort study. Say we look at people with lung cancer and those without, then go back in
their history and figure out who was a smoker. That is a case-control study. All could be shown on a 2x2 table.
* Odds means ratio by definition. Odds of having lung cancer is a/c then not having lung cancer is b/d. The odds
ratio is the ratio of those odds, (a/c) / (b/d) = (a*d) / (b*c).
* The 2x2 from left to right, top to bottom is a b c d. A study about smokers and lung cancer would have the “a” cell
as smokers who have lung cancer. “a” cell is always disease with risk factor. “d” cell is diagonal from “a”.
* Odds ratio mnemonic is AD (present time, like 2000AD) over BC (past time, dinosaurs in BC time).
* Odds ratio is sometimes read “smokers are 9.5x more likely to get lung cancer” but it should be read “people with
lung cancer are 9.5x more likely to have been smokers.” You did not start with smoking and predicted lung cancer,
you started with lung cancer then went back and found smokers.
Research Designs: Experimental Studies
* Research in medicine that is experimental is labeled clinical trials.
* To see if a drug makes people better, we should give it to people and see if they get better? Wrong; we have to
compare that group to a group of people who did not get the drug and see if those people get better.
* A control group is a group of people who get everything except for the intervention being investigated.
* The three phases of clinical trials are what the U.S. Food and Drug Administration (FDA) requires a drug to pass
so that they can be prescribed in medical practice.
* Phase I tests how well the drug performs with healthy volunteers. Does it harm or not harm healthy people?
* Phase II is the first time we use real patients, a small group of them. We are trying to figure out if the drugs works
to a point, but really we care about the correct dosage for the drug, what the protocol should be for giving the drug.
We get some outcome data but the goal is to get information to setup for the Phase III trial.
* Phase III trial is where the clear decision is made about whether the drug works and if we should be using it on
patients that have the particular disease. It is characterized by a large group of patients because we are looking for
efficacy (how well does the drug work) and side effects. You won’t find side effects emerging with a small group of
patients. If a drug passes a Phase III trial, meaning we get statistical significance relative to a control group, then the
drug is available for the FDA to certify and we can prescribe it in clinical practice.

DO NOT DISTRIBUTE - 43 -
Study Notes – Family Medicine & Physical ExamJames Lamberg 28Jul2010

* Phase IV clinical trial is when the drug is being prescribed but we are keeping track of what is happening. It is not
really a clinical trial per-se but more record-keeping of what is happening.
* Randomized controlled clinical trial (RCT) is the verging on the best kind of research. Randomized means
whether the patient ends up in the intervention or the control side is completely up to chance.
* Say you setup a trial about antidepressants. For each person that comes into your clinic, you assign them to one
group, the next person to the other group, and back-and-forth. Is this randomized? No. To be randomized means any
person has an equal chance of being in either group and all combinations are possible. So if you walk in behind
someone then you would not end up in the same group, however being in the same group has to be a possibility for
random assignment.
* A better way to randomly assign people would be to shuffle a deck of cards then flip a card for each person. If the
card is red they go to group A and black they go to group B. Theoretically the first 26 people could be in the same
group. That is a random assignment.
* Double-blind randomized clinical trial is the best type of research we can do. Double blind means the subjects do
not know if they are getting the intervention or in the control group. Also, the researchers who have contact with the
subjects do not know if they are in the intervention or the control group. So both the patients and researchers cannot
be biased, the patients do not respond differently and the researchers do not treat the patients differently.
* True or False: in a double-blind randomized clinical trial nobody knows who is in the intervention or the control
group. False, somebody has to know. The true statement is that the subjects and the researchers who have contact
with the subjects do not know. Someone, like a lab technician or computer analyst, has a coded list.
* If RCT are the gold standard, why don’t we just run those all the time? Well, because they are expensive and time
consuming. So we focus our resources on things that are likely to show good outcomes. That is why we do all the
observational studies ahead of time, that way we get good evidence that it is useful to move on into the experimental
studies.
* Say we have a drug that is likely to add a few years to a patient’s life. We could give the drug to all those with the
disease, but that wouldn’t be good research. We have a conflict of what we need from research and what we can
tolerate with ethics. The solution is a crossover design study.
* Crossover study starts with two double-blind randomized groups. At the beginning, group A gets the real drug and
group B gets a placebo that looks like the real drug. At some point, unknown to the patients or researchers, we
switch the groups so that group A is now getting the placebo and group B is getting the real drug. In this study,
every patient in the study gets the drug at some point thus we have no untreated patients. We also have a comparison
between the groups at each point and we can compare the individuals themselves when they are on or off the drug.
So we have good research design and research ethics satisfied.
--------------------------------------------------------------------------------------------------------------------------------------------
Kaplan Videos (2001) – Biostatistics with Dr. Steven Daugherty, MD
--------------------------------------------------------------------------------------------------------------------------------------------
Descriptive Statistics
* Central tendency is measured by the mean, median, mode. Mean is the average. Median is the middle number, half
numbers above and half below. Mode is the most common number, what you have the most of.
* In a normal distribution, the mean equals the median equals the mode.
* Positive skew is when the tail of the curve points to the right. Negative skew is when the tail points to the left.
* Positive skew: mean > median > mode. Start at tail an go inward in that order, alphabetical order.
* In a skewed distribution the mean is most sensitive to extreme values, so it is pulled toward the tail. The mode is
the least sensitive to extreme values.
* Negative skew: mean < median < mode. Again, start at the tail and go inward. The mean is pulled most toward the
extreme values.
* You can tell if a distribution is skewed by comparing the mean to the median. If the mean > median, we have a
positive skew. If mean < median, we have a negative skew. If mean = median, we have a normal distribution.
* For most normal distributions, the measure of central tendency that we use is the mean because it is the most
intuitive for people. For a skewed distribution the mean is not the best measure, use the median as the measure for
central tendency.
* When looking at salary (skewed distribution), you would want to use the median. Say most electrical engineers ear
$75,000/year but a few guys earn $2 billion/year. That would really skew the mean, but the median would be closer
to the actual salary that you would earn.
* Now that we have central tendency, we want to know about the spread. How wide or narrow is the distribution?
* The most common and intuitive measure of spread is range, which is the highest - lowest score. It is a poor
measure of the spread however. The range ignores most of the data we have, concentrating on 2 numbers.

DO NOT DISTRIBUTE - 44 -
Study Notes – Family Medicine & Physical ExamJames Lamberg 28Jul2010

* Suppose there is an exam today and a similar test next week. Your score is likely to shift between the two tests.
Your score is going to shift in a predictable direction if you are at the extremes, they shift toward the mean. This is
called regression towards the mean. It means that extreme values when re-measured will be less extreme.
* Say you took Step 1 and got a really high score. You are more likely to get a lower score on Step 2, not because
you got dumber, but because of regression toward the mean.
* Suppose you have a patient that has the highest blood pressure you have ever seen in your clinical career. You leap
into action, give them strong medication, get them to exercise, put them on a diet, and schedule to see them again
one week later. Now one week later, the blood pressure is substantially lower than before. What accounts for this
reduction in blood pressure? Not the drugs, not the exercise, not the diet, it is regression toward the mean. It is
simply an artifact of measurement.
* Since the extreme score are the least stable of all the scores, we can see that the range is a poor measure of spread.
* So how should we measure spread? Lets take each score (x) and subtract the mean (xbar) and sum them. This
shows us how close or far from the mean each score is, so we get the average deviation from the mean. This sounds
good on paper but doesn’t work well in practice because we always get zero. For each number above the mean there
is a similar number just as far below the mean. If we square that match, we don’t get numbers zeroing out. But now
we have to account for the number of people, so we divide by the sample size - 1. Don’t worry about the -1, it is a
degree of freedom and just adjusts for the mean.
* Standard Deviation = sqrt((SUM (x - xbar)^2) / (N - 1))
* Yes, it looks like a complicated formula, but we did the talk so you understand why the formula was created.
* As the sample size increases, the standard deviation increases, true of false? False, the standard deviation should
not change with sample size. We always divide by the sample size in the formula.

* In a normal distribution, what percentage of cases fall within +/- 1 standard deviation? 68%.
* Memorized these numbers: +/-1 SD = 68%. +/- 2 SD = 95.5%. +/-3 SD = 99.7%.
* What percentage of scores fall between 0 and +1 SD? 34%. We calculate that by dividing +/- 1 SD (68%) by 2.
* Say the average for a test is 500 and standard deviation is 100 (e.g. COMLEX). If you scored 600, what percentile
are you? Here you are +1 SD above the mean. That means you are 34% above the mean (50%), which is 84%ile.
This means you scored better than 84% of everyone taking the test and only worse than 16%.
* What percentage of cases in a normal distribution fall above one standard deviation below the mean? That would
be 34% + 50% or 84%.
* What percentage of cases in a normal distribution fall below two standard deviations below the mean? This is
roughly 2.25%. We know that +/- 2 SD is 95.5% leaving 4.5% leftover on each tail. We only need half of that.
* IQ Distribution: Average = 100, Standard Deviation = 15.
* What percentage of the population has an IQ below 70? 2.25%. What about > 130? 2.25%.

DO NOT DISTRIBUTE - 45 -
Study Notes – Family Medicine & Physical ExamJames Lamberg 28Jul2010

* Say USMLE Step 2 has a theoretical mean of 200 and standard deviation of 20. If you score a 240 on the exam,
what percentile are you at? You are at the 97.75%ile. What if you score 180? That is 16%ile.
* Say that 95% of people scored on a test between 150 and 250 on a test given to the general population. What is the
84%ile for this test? Can we assume this is a normal distribution if the test is given to the general population? Sure.
So we know +/-2 SD of people are between 150 and 250, so the mean must be 200 in the middle. To get to the
84%ile we need mean + 1SD. If 2 SD = 50 then 1 SD = 25, so 84%ile is 200 + 25 = 225.
* What would you estimate the 16%ile to be in the previous case? That is the mean -1 SD, so 200 - 25 = 175.
Inferential Statistics
* Inferential statistics are taking some information and inferring from that information what the whole rest of the
world must be like. It is because of inferential statistics that we can do science and research or decide what drugs
you should or should not be giving your patients. You make a leap from a study on a group of people and infer how
the drug would work on anyone getting it.
* Say you have a class of 200 students and want to know how they did as a whole. But, you’re in a hurry so you
randomly take 6 students and calculate the mean. Will that mean be exactly the same mean as the whole class?
Doubtful. But, is it likely to be close? Sure.
* With inferential statistics we give up precision (knowing the exact number) and get a degree of confidence or
certainty about what the exact number is probably.
* Confidence intervals are how we do research in medicine and you have to understand confidence intervals to
understand basically any research article you get.
* Say class starts at 8am. But you come strolling in 5mins late but you think you’re on time. You know the number
isn’t exact but you’re in the ballpark.
* Confidence Interval (CI) = Xbar +/- Z (S / sqrt(N))
* The part in the parenthesis is called the standard error (SE). So do you want this to be a large number or small
number? Small number, you want the error to be as low as possible. The SE is essential a measure of the quality of
your estimate. S is the standard deviation and N is the number of people.
* The smaller the standard deviation the better your estimate, the larger the SD the worse the estimate. As the
sample size gets larger the standard error gets smaller. Thus, we put SD on top and N on the bottom for SE.
* The one thing you can modify for standard error is the sample size. This is your way to reduce the error.
* Say a group of obstetricians wants to do a new study comparing a new birthing method with the current method.
You ask what the sample size is and they say 4. Is this study doing to reach significance? No way, the reality you get
from a sample size of 4 is so skewed compared with the general public that you won’t be able to make an inference.
* Z is the standard score or Z-score. Read carefully here, there is a standard deviation, standard error, standard score.
* Standard score (Z) implies the mean (Xbar) = 0 and SD = 1 because these numbers are easy to work with. So if the
number is positive it is on the right side of the mean, and negative is on the left side of the mean. SD of 1 means that
the Z-score is in standard deviation units.
* If your Z-score is +1, you are 1 SD above the mean. If your Z-score is -1.6, you are 1.6 SDs below the mean.
* Say we want 95% of our scores within 2 SD of the mean. We could go +/- 2 for our Z-score, but that’s 95.5%. To
get 95%, we tuck it in to Z-score = +/- 1.96. 99% of cases for Z-score = +/-2.58.
* Z = 1.96 for a 95% confidence interval. Z = 2.58 for a 99% confidence interval.
* When you’re in the hurry during the middle of an exam, use approximates Z = 2 for 9% and Z = 2.5 for 99%.
* Say Xbar = 60%, S = 9%, and N= 9. What is the 95% CI for that estimate of the mean? CI = 60 +/- 2 (9/sqrt(9)),
which is 60 +/- 6. This means if you got all the data and not just the 9 people, you would likely find the real mean to
be between 54 and 66. Are we completely certain the mean is between 54 and 66? No, we’re 95% certain. Is the
mean most likely to be 60 in the middle? No, it is just as likely to be 54 as it is 60 or 66. This is a flat probability
distribution, so any number inside is as good as any other number in the confidence interval.
* Confidence interval is our window to reality. We have a set of data representing a small group and we want to
infer what that data would look like for everyone, so the confidence interval is our best guess.
* Say 187 people are polled regarding who they want the next president to be, AG or W. AG gets 38% and W gets
44%. The margin of error is 3% for the poll (assume this is 95% CI). Can you say with 95% confidence that W is
leading? So AG is as high as 41% and as low as 35%, and W is as high as 47% and as low as 41%. They could both
be 41%, so no you cannot say that W is leading the poll because there is overlap.
* Rule: Any overlap in a confidence interval means no difference.
* It would be better to choose a drug that has a more predictable response pattern than a drug that has better effect
but less predictable response. So look for a smaller standard deviation to get the more predictable response.
* How do you determine if a relative risk number is significant? Look at the confidence interval. The critical number
is 1.0. The relative risk will always occur within the confidence interval given. See if the confidence interval

DO NOT DISTRIBUTE - 46 -
Study Notes – Family Medicine & Physical ExamJames Lamberg 28Jul2010

contains the number 1.0. If the confidence interval contains 1.0, it is not significant. If the interval is outside 1.0,
then it is significant. Why is 1.0 the critical value? We divide to get the relative risk, so if the risk in group A is the
same as the risk in group B we would get a relative risk of 1, meaning the groups are the same. To prove they are
different, show the outcome of the relative risk is outside of 1.
* A relative risk > 1 with confidence interval not including 1 indicates increased risk.
* A relative risk < 1 with confidence interval not including 1 indicates decreased risk.
* The relative risk does not matter unless the confidence interval matters, so make sure the CI does not include 1.
* This is all the same for odds ratio. You can tell if an odds ratio is significant if the CI does not include 1.
Statistical Probability
* You flip a coin 4 times and it comes up heads every time. If you think the next flip will more likely be tails since
you are due for a tails, you fell for the gambler’s fallacy. What came before has nothing to do with what will happen
because the coin flips are independent events.
* What is the chance of getting two heads in a row (joint probability)? We multiple. Probability of heads and heads,
the and means multiply. Chance of getting heads is 50%, so 50% times 50% is 25% change of 2 heads in a row.
* What is the chance of getting 4 heads in a row? (50%)^4.
* You flip a coin and it came up heads. What do you know? You know the coin came up heads and that it did not
come up tails. On a single event, heads and tails are mutually exclusive. If one occurs the other cannot occur.
* What is the chance of getting heads or tails on a single flip? Or means add. 50% for heads + 50% for tails is 100%
chance of getting either heads or tails on a single flip.
* Mutually exclusive in Venn diagram would be two circles that do not overlap. Non-mutually exclusive in Venn
diagram would be two overlapping circles, you can be part of both circles at the same time.
* Joint probability of circle A and B if they are mutually exclusive is A + B. What about when you have non-
mutually exclusive? If you add A + B you counted the overlap twice, once for A and once for B. The overlap area is
A times B, so this is the correction factor. The formula then for overlapping circles is A + B - A*B, the chance of
getting A, B, or both. What is the chance of getting A or B but not both? A + B - 2*A*B.
* Say the chance of meeting someone who is obese is 30% and the chance of meeting someone with diabetes is
10%. What is the chance of meeting someone who is either obese or has diabetes, but not both? Using A + B -
2*A*B, we get 30% + 10% - 2*(30%*10%) = 40% - 6% = 34%. What is the chance of meeting someone who is
obese, or someone who has diabetes, or someone who is obese with diabetes? 37%.
* What is life expectancy following surgery? You are being asked about the median survival time, or when 50% of
people have died.
* For a given surgery, say 90% live to one year, 75% live to two years, and 50% live to three years. The life
expectancy following surgery is thus 3 years (50% mark). If you live to two years, what is the chance that you will
make it to three years? 50% / 75% = 2/3 = 67%. This is a conditional probability problem, similar to the question
about two trains heading toward each other and asking when they will meet.
Statistical Logic
* The problem with statistics is that most people think instinctually the exact opposite of what the statistics.
* You cannot prove something with logic. You can’t say that a drug works. But, you can disprove something so you
can disprove that the drug doesn’t work. Sounds pretty backwards but that’s how it works.
* Say there is a new anti-diarrhea medication and we want to research if it works. So, we take a group of people with
explosive diarrhea and randomize them to either control or the group that gets the medication. One group gets the
pill and the other gets the placebo but neither the participants nor researchers contacting the patients know which
pill. Now, we have to lay out the null hypothesis.
* The null hypothesis is the opposite of what we want to find. If we want to know if the drug works, the null
hypothesis is that the drug does not work. Don’t worry about one-tailed versus two-tailed null hypothesis.
* For our study, the null hypothesis would be that the group getting the anti-diarrhea medication fails to resolve their
diarrhea before the control group.
* Data gets fed into a computer. What happens in the computer? Who cares for now. The output will be some letters,
like t (t-test), F (F-test), r (correlation coefficient), or x^2 (chi squared). Just ignore those for the exam, honestly.
The one letter you should focus on is p (p-value). This is your key value for making statistical decisions.
* The p-value is both the standard and a summary of the empirical evidence that you compare to the standard.
* The p-value is set before the research begins. When you aren’t sure what to do, you do what everyone else does
which is p ≤ 0.05. For a 95% CI, you are right 95% of the time. The 5% is the percent of the time that you are
wrong, so it corresponds exactly to p ≤ 0.05.
* If p = 0.02, are we under or over the bar? We are under the bar, which is good. This means we can reject the null
hypothesis. We want to reject the null hypothesis because it is the opposite of what we are looking for.

DO NOT DISTRIBUTE - 47 -
Study Notes – Family Medicine & Physical ExamJames Lamberg 28Jul2010

* Is it possible that in our study the drug works but it won’t work for the rest of the world? Not likely but possible.
So you have to realize that the decision you make here could be wrong, a Type I or alpha error.
* Type I Error means you rejected the null hypothesis but you should not have. You decided that the drug works but
in reality it is garbage. You will never know for sure that you made a Type I error, but you will know the chance or
probability that you made a Type I error. The chance that you made a Type I error is the p-value.
* If p = 0.02, there is a 2% chance of a Type I error. So out of 100 times we will be wrong twice. But, this is how
statistics and medicine work. We have to go ahead and keep making decisions anyhow.
* Type I Error (alpha error) = p-value, the alpha criterion.
* If p = 0.13, we are above the bar so we cannot reject the null hypothesis. Should we accept the null hypothesis?
No. We fail to reject the null hypothesis. Never accept the null hypothesis. The same holds for court where the jury
does not find you innocent, they say there is not enough evidence to convict you.
* If p=0.13, can we have a Type I error? No because you have to reject the null hypothesis to get a Type I error.
* Type II error is when you did not reject the null hypothesis but you should have. In your study the drug was
garbage but in the real world the drug is fantastic. What is the chance that you made a Type II error? You don’t
know and there is no information to tell you.
* Type II Error (beta error) = you don’t know.
* In general, which is worse, a Type I error or a Type II error? A Type I error, which is why the p-value is so
important. A fault of commission is something you did that you shouldn’t have done. A fault of omission is
something you didn’t do that you should have done. It’s considered worse in general to fault with commission.
* It is worse to give patients a drug that will not help them (and might hurt them) than to fail to find a drug that
would help them. Primum non nocere = first do no harm.
* You are given a drug with p = 0.02 in clinical research. A patient comes in and you prescribe the drug. The patient
asks you what the chances are that the drug will work for them. The answer does not deal with the p-value. It does
not tell you the percentage of patients who will benefit, the chance your patient will benefit, or the chance that side
effects happen. You can look in the research article for a table that shows how many people got better when given
the drug along with how many people did not get better when given the drug.
* Power is capacity to find a difference if there is a difference to find.
* Beta = 1 - Power = Type II Error
* What is the simplest thing you can do to increase statistical power? Increase sample size, N.
* A researcher found p=0.15 with statistical power of 70%. What is the chance of a Type II error? 30%.
Statistical Analysis
* The type of statistical test that you run depends on the type of data you have. The world is not made of numbers, it
is made of things. We have to convert those things into numbers and we do that by scaling. Once you recognize the
type of scale you have, you know the type of statistical analysis you should do.
* Nominal or Categorical data is grouping things into clusters. The most common nominal variable is gender, male
and female. So male and female, how many nominal variables? One, gender. Gender is a single nominal variable
with two groups.
* Interval data is along a scale, where things are measured at equally graded intervals. From interval data we can get
means and standard deviation. Can we computer the mean of gender, not so much. How about the mean of the
height of students in a class, yea.
* When you’re talking about what something is, that is nominal data. When you’re talking about how much of
something you have, that is interval data.
* Pearson correlation is for interval data with two variables (most likely).
* Chi squared is for all nominal data (most likely).
* t-test for nominal and interval data combined (most likely).
* If you want to compare the heights of men and women. What statistical test should we use? We have men and
women, which is nominal data, and height, which is interval data. So t-test.
* If you want to compare height to weight, how would you do that? You have two types of interval data so do a
Pearson correlation.
* Say you want to do a simple study where one group gets a drug and the other group does not. Then see who is
better or not better at the end of the year. How would you analyze that data? You have all nominal data so do a chi
squared analysis.
* Correlation coefficient (R) ranges from a possible -1 to +1. The closer to +1 or -1 the better. The cleaner the line
or trend you have on a graph the stronger the correlation. Positive correlation is direct and negative correlation is
inverse. Would you expect a positive or negative correlation between height and weight? Positive. How about
number of minutes exercised per week and blood pressure? Negative.

DO NOT DISTRIBUTE - 48 -
Study Notes – Family Medicine & Physical ExamJames Lamberg 28Jul2010

* A t-test only works when you have two groups. Say you want to compare the heights of three different animals,
then you cannot do a t-test. Mnemonic is “t for two”. If you have more than two groups you do an analysis of
variance (ANOVA).
* Chi squared is for all nominal data so you will likely see a table, like a 2x2 or 3x3.
* Again, Pearson correlation is for interval data, chi squared is for nominal data, and t-test is for nominal and
interval data combined with two groups. If you have more than two groups, use ANOVA.
--------------------------------------------------------------------------------------------------------------------------------------------

DO NOT DISTRIBUTE - 49 -

You might also like